Sei sulla pagina 1di 51

ADrPlexus – Digital Strides 2017

NIMHANS MOCK 3
1. A 45-year-old man presented with diplopia, In kwashiorkor (rarely in marasmus) the small bowel
dysarthria and difficulty with swallowing. Over the shows a decrease in the mitotic index in the crypts of the
next few days he developed weakness of the upper and glands, associated with mucosal atrophy and loss of villi
lower limbs. On day 4 he was unable to walk unaided. and microvilli. In such cases concurrent loss of small
He denied any sensory symptoms or bladder intestinal enzymes occurs, most often manifested as
disturbances. His previous medical history is disaccharidase deficiency. Hence, infants with kwashiorkor
unremarkable. He is a non-smoker, does not drink initially may not respond well to full-strength, milk-based
alcohol excessively. He does not take any drugs On diets. With treatment, the mucosal changes are reversible.
examination he was apyrexial. His general medical The bone marrow in both kwashiorkor and marasmus may
examination was normal. His higher mental function be hypoplastic, mainly as a result of decreased numbers of
was unremarkable. There were no signs of meningism. red cell precursors. The peripheral blood commonly
Cranial nerve examination showed bilateral dilated reveals mild to moderate anemia, which often has a
and fixed pupils. He had binocular diplopia but no multifactorial origin; nutritional deficiencies of iron, folate,
obvious ophthalmoplegia. He was dysarthric with and protein, as well as the suppressive effects of infection
weak cough. His vital capacity was 3.15 standing and (anemia of chronic disease) may all contribute. Depending
2.00 lying flat. He had lower motor neuron on the predominant factor, the red cells may be microcytic,
tetraparesis of power 3/5. He was hyporeflexic with normocytic, or macrocytic.
normal sensation. He was unable to walk unaided. The brain in infants who are born to malnourished
Blood tests including FBC, U+Es, LFTs, TFTs, Ca, mothers and who suffer PEM during the first 1 or 2 years
Autoantibody screen, ESR, CRP were normal. ECG and of life has been reported by some to show cerebral
CXR were unremarkable. CT brain was normal. Nerve atrophy, a reduced number of neurons, and impaired
conduction studies and EMG were normal. What is the myelinization of white matter.
most likely diagnosis? Many other changes may be present, including (1) thymic
1. Guillain Barre Syndrome and lymphoid atrophy (more marked in kwashiorkor than
2. Lyme disease in marasmus), (2) anatomic alterations induced by
3. Myasthenia gravis intercurrent infections, particularly with all manner of
4. Botulism endemic worms and other parasites, and (3) deficiencies of
other required nutrients such as iodine and vitamins.

Ans. 4 3. The most common form of vascular ectasia is


The clinical presentation of descending weakness with 1. Nevus Flammeus
autonomic dysfunction (fixed dilated pupils) is typical of 2. Spider Telangiectasia
botulism. It is a neuromuscular junction disorder and 3. Osler Weber Rendu disease
therefore nerve conduction studies and EMG are normal. 4. Bacillary Angiomatosis
Repetitive nerve stimulation showing incremental
responses, which is diagnostic of Botulism. CSF analysis is Ans. 1
usually normal. Nevus Flammeus
Robbin’s 8th edition Page no 520
2. The following are features of Marasmus except Vascular ectasias
1. Fatty liver Nevus flammeus
2. Hypoplastic bone marrow Spider telangiectasia (arterial spider)
3. Thymic atrophy Hereditary hemorrhagic telangiectasis (Osler-Weber-
4. Lymphoid atrophy Rendu disease
Reactive vascular proliferations
Ans. 1 Bacillary angiomatosis
fatty liver Nevus flammeus is the ordinary “birthmark” and is the
Robbin’s 8th edition Page no 429 most common form of ectasia; it is characteristically a flat
The central anatomic changes in PEM are lesion on the head or neck, ranging in color from light pink
(1) growth failure, to deep purple. Histologically, there is only vascular
(2) peripheral edema in kwashiorkor, and dilation; most ultimately regress.
(3) loss of body fat and atrophy of muscle, more marked in The so-called port wine stain is a special form of nevus
marasmus. flammeus; these lesions tend to grow with a child, thicken
The liver in kwashiorkor, but not in marasmus, is enlarged the skin surface, and demonstrate no tendency to fade.
and fatty; superimposed cirrhosis is rare. Such lesions in a trigeminal nerve distribution are
1
occasionally associated with the Sturge-Weber syndrome Investigations of choice in respiratory system
(also called encephalotrigeminal angiomatosis). Sturge- Disease Investigation
Weber syndrome is a rare congenital disorder associated o Intersitial lung disease HRCT
with venous angiomatous masses in the cortical o Bronchiectasis HRCT (1stchoice)
leptomeninges and ipsilateral facial port wine nevi; mental Bronchography (2nd
retardation, seizures, hemiplegia, and skull radioopacities choice)
also occur. o Pulmonary embolism CT with i.v. Contrast (1st
choice)
4. Compliment dependent cytolysis is an example of o Solitary pulmonary V/Q scan (2nd choice)
which type of HSR nodule
1. Type I HSR o Calcification CT scan
2. Type IV HSR o Pulmnnary malignancy CT scan
3. Type III HSR o Superior sulcus or CT scan (except in superior
4. Type II HSR pancoast tumor sulcus or pancoast tumor)
MRI
Ans. 4
TYPE II HSN : 7. The diagnostic test based on Antigen – Antibody
1.Compliment depd cytolysis – Transfusion reaction, reaction which result in insoluble precipitate
erythroblastosis fetalis, AI hemolytic anemia 1. WIDAL
2.Compliment depd inflammation (C3a,C5a) – GoodPasteur 2. Lancefield grouping (Streptococcus)
syndrome, Pemphigus vulgaris, Bullous pemphigous, 3. Tuberculin test
rheumatic fever , vasculitis 4. Ames test
3. Compliment depd phagocytosis
Ans. 1
5. Succinylcholine is used with caution in all Antigen Antibody Reaction
except - Precipitation –
1. Mysthenia gravis  Definition- {Soluble Antigen + Antibody} → insoluble
2. Burns precipitate / floccules
3. Crush injury  Ring test –
4. Tachycardia o Ascoli thermo precipitation test ( anthrax) ,
o Lancefield grouping (Streptococcus)
Ans. Is '4' i.e.. Tachycardia  Slide flocculation test – VDRL, RPR
 Sch is the only muscle relaxant that causes vagal  Tube flocculation test – Kahn test, standardization of
stimulation  Causes bradycardia, (attenuates toxin
tachycardia & hypertension).  Immuno-diffusion ( In gel) –
 In bums, crush injury and mysthenia gravis can cause o Elek gel precipitation (C diphtheriae toxigenicity
severe hyperkalemia, testing),
 Succinylcholine can cause dangerous hyperkalemia in o Eiken(E.coli)
patients with; -
□ Burn □ Rhabdomyolysis 8. Ritter’s disease is caused by
□ Massive trauma □ Severe Parkinsonism 1. Pseudomonas auroginosa
□ Crush injury □ Tetanus 2. Streptococcus pneumonia
□ Severe intra-abdominal infection □ Polyneuropathy 3. Bacillus anthracis
□ Spinal cord injury □ Closed head injury 4. Staphylococcus aureus
□ Hemiplegia/Paraplegia □ Stroke
□ Muscular dystrophy □ Encephalitis Ans. 4
□ GB syndrome □ Mysthenia gravis Epidermolytic toxin (Exfoliative toxin) by Staphylococcus
responsible for Ritter disease. Epidermolytic toxin
6. Best imaging modality in bronchogenic carcinoma (Exfoliative toxin) Mainly belong to phage group II
1. CT Two types of toxin : A: heat stable, chromosomal gene, B:
2. MRI heat labile, Plasmid controlled
3. Bronchoscopy Presentation: Scalded skin syndrome, Severe- Ritter
4. None disease (newborn), TEN (adult)
Milder- Pemphigus neonatorum, bullous impetigo
Ans. is ‘1’ i.e., CT
 CT scan is the investigation of choice for pulmonary 9. Which test is performed to detect reversible
malignancy except for superior sulcus or pancoast myocardial ischemia?
tumor, where MRI is the investigation of choice. 1. Coronary angiography

2
2. MUGA scan
3. Thallium scan 11. Neurofibroma is -
4. Resting echocardiography 1. Autosomal dominant
2. Autosomal recessive
Ans. is ‘3' i.e., Thallium scan 3. X-linked recessive
 Thallium is used for perfusion scan to sec the viability 4. X-linked dominant
(reversibility) of myocardium.
Radionuclide scanning Ans. is '2' i.e., Autosomal recessive
 There are two main forms of cardiac scanning with  Neurofibromatosis comprises of two distinct
radionuclides: - A) Myocardial imaging; and B) Nuclear disorders
angiography o Neurofibromatosis I
A) Myocardial imaging: - Myocardial imaging can be o Neurofibromatosis II
achieved in two ways: -  The genes for these are located on different
1) Infarct scanning: - Uses an isotope (pyrophosphate) chromosomes.
which accumulates in damaged myocardium. The Area  Both are inherited in an autosomal dominant pattern.
of infarct appears as Hot spot. o NF-1: Neurofibroma gene on chromosome 17.
2)Perfusion scanning:- Perfusion scan looks at the blood o NF-2; Neurofibroma gene on chromosome 22.
(lowing through the coronary arteries and is useful Neurofibromatosis type I (Von-Recklinghewsen
in the diagnosis of coronary artery disease. It can also be disease)
used to identify areas of reversibility following a o NF-1 is diagnosed when any two of the following
MI. A radioisotope, most commonly thallium is injected seven signs are present.
into a peripheral vein. It travels to the coronary 1. Six or more cafe-au-lait macules
arteries and images arc taken with a special gamma o >5 mm in prepupertal individuals
camera of the heart. Other isotopes used tor perfusion o >15 mm in postpuberal individuals
scanning are technetium and M1B1 (2-mcthoxy Iso-Butyl o Cafe-au-lait spots are the hallmark of
Isonilrite). The area appears as cold spot because neurofibromatosis and are present in almost
radionuclide binds to viable area. 100% of the patient.
B) Nuclear angiography: - Nuclear angiography can also 2. Axillary or inguinal freckling
be done on two different ways: - 3. Two or more Lisch nodules.
1) First pass technique: - It involves rapid i.v. injection of o Lisch nodules are hamartomas located within the
a bolus of a simple radionuclide (Tc-99 pertechnetate). iris.
Its passage through the cardiac chambers is then recorded. 4. Two or more neurofibroma or one plexiform
The method is most useful for the study of intracardiac neurofibroma.
shunts. o Typically involve the skin, but may be situated
2. Multigated equilibrium studies (MUGA): - A along peripheral nerves and blood vessels.
radioactive isotope (technetium) is injected into a o They are small, rubbery lesions with a slight
peripheral vein. The gamma camera is able to detect purplish discoloration of the overlying skin.
technetium labelled RBC and produces moving images of a 5. A distinctive osseous lesion.
heart.A MUGA scan can assess the degree of damage to the o Sphenoid dysplasia or cortical thinning of long
myocardium following a ML More importantly, it can bones.
give an accurate and reproducible assessment of left 6. Optic glioma
ventricular function. It is used in patients undergoing 7. A first degree relative with NF-I
chemotherapy (e.g., anthracyclin) to assess whether the o Other findings are: -
drugs are having an adverse effect on cardiac function. o Pseudoarthrosis of tibia.
o Scoliosis is the most common orthopaedic
10. Pfeiffer’s bacillus is problem in NF-1, but is not specific enough to be
1. Haemophillus infuenzae included as a diagnostic criterian.
2. Haemmophihlus aegypticis o Short stature
3. C.diphtheriae o Mental retardation, epilepsy
4. Burkholderia pseudomallei o Hypertension
o Aqueductal stenosis with hydrocephalus
Ans. 1 o Meningiomas, ependynomas, Astrocytomas,
phcochromocytomas.
 Kleb-Loeffler bacilli– C.diphtheriae o NF-I is caused by mutation in NF-1 gene on
 Preisz Nocard bacilli – C.pseudotuberculosis chromosome 17 which encodes protein
 Koch Week bacilli – H.aegypticis neurofibroma I.
 Whitmore bacilli – Burkholderia pseudomallei Neurofibromatosis type-2
 Pfeiffer’s bacilli – Haemophillus infuenzae

3
o NF-2 may be diagnosed when one of the following 4. Madurellosis
two features arc present.
1. Bilateral ocoustic neuroma - Most distinctive feature Ans. 3
2. A parent, sibling or child with NF-2 and cither Nocardia
unilateral eighth nerve masses or any two of the
 Strictly aerobic , acid fast, urease+ve,
following Neurofibroma, meningioma, glioma,
Schwannoma or juvenile post subcapsular cataract. exogenous infection
NF-2 is cause by mutation in NF-2 gene on  Nocardia – grow on Sabouraud’s dextrose
chromosome 22 that encodes for agar, brain heart infusion agar
proteinneurofibromin 2, Schwannomin or merlin.  For isolation of Nocardia from soil, paraffin
bait technique used.
12. Most common benign heart tumor is-
1. Rhabdomyoma 14. True statement about clostridium perfringens all
2. Hemangioma except
3. Lipoma 1. It is the commonest cause of Gas gangrene
4. Myxoma 2. It can be detected by Nagler's reaction
3. The food poisoning spores of clostridium perfringens
Ans. is ‘4’ i.e., Myxoma are heat sensitive
o The most frequent cardiac tumor for all age groups is - 4. The major enzyme secreted is hyaluronidase
Metastasis.
o The most frequent primary cardiac tumor for all age Ans. 4
groups is - Myxoma. Hyaluronidase( "spreading factor" ) -Breaks down
o The most frequent primary cardiac tumor in infants hyaluronic acid, enabling the bacteria to spread between
and children is - Rhabdomyoma. cells produced by Staphylococcus aureus and streptococci.
Cardiac myxoma Enzyme responsible for gas gangrene in C.perfringens will
o This is the most common primary tumor of heart be lecithinase
o Most commonly seen in left atrium.
o More common in females. 15. Not a feature of HELLP syndrome -
o Myxoma may be of two types 1. Haemolysis
1. Sporadic myxomas 2. Elevated liver enzymes
o These are the usual form of myxomas (more than 90% 3. Low platelet count
of myxomas are sporadic) 4. Renal failure
o Mean age is 56 years.
o Most sporadic myxomas aresolitary. Ans. is ‘4’ i.e., Renal failure
2. Familial myxomas HELLP syndrome
o Constitute less than 10% of all myxomas. o It is an acronym for Haemolysis(H), Elevated liver
Occurs at younger age. enzyme(E) (AST & ALT> 70IU/L, LDH> 600IU/L), Low
o Myxomas are bilateral and multiple. platelet cout(L)(<100000/mm3).
o Myxomas may occur in locations other than atria. o It is a rare complication of pre-eclampsia but may
o Some patients with cardiac myxoma have a syndrome, develop even without hypertension.
i.e. carney syndrome that consists of: - o Manifested by nausea, vomiting, epigastric or right
i) Myxoma ii) Spotty pigmentation upper quadrant pain.
iii) Endocrinopathy
Morphology 16. Dohle bodies are seen in -
o The region of the fossa ovalis in the atrial spetum is 1. Multiple myeloma
the favored site of origin. 2. May-Hegglin anomaly
o Myxoma may be pedunculated or sessile. 3. Waldenstorm Macroglobulinemia
o Pedunculated myxoma can move and obstruct the AY 4. Lymphoma
valve.
o Myxomas are gelatinous structures consisting of Ans. is '2' i.e., May-Hegglin anomaly
myxoma cells (lepidic cells), endothelial and smooth Dohle bodies
muscle cells embedded in a stroma rich in  Dohle bodies are light blue gray, bosophilic inclusions
glycosamincglycans. in the peripheral cytoplasm of neutrophils,
 They are thought to be remnants of the rough
13. Paraffin baiting mixed culture is done for endoplasmic reticulum containing RNA.
1. Botryomycosis  They are seen in: Burns, Infections, Physical trauma,
2. Actinomyces Neoplastic diseases, Wissler's disease, May-Hegglin
3. Nocardiosis anomaly, Chcdiak-Steinbrinck-Higashi's syndrome.

4
extensive, diffuse fibrosis throughout the thyroid gland,
17. The following are true of ovarian choriocarcinoma often associated with a prominent lymphocytic infiltrate,
except simulating Hashimoto thyroiditis. Lymph node metastases
1. Are usually found in association with other germ cell are present in almost all cases. The papillary
tumours microcarcinoma, which is defined as an otherwise
2. Is extra-embryonic differentiation of malignant germ conventional papillary carcinoma less than 1 cm in size,
cells. and usually confined to the thyroid gland.
3. Ovarian primaries are indolent tumours
4. They elaborate high levels of chorionic gonadotropins 19. Elution mechanism due to
1. Haemagglutinin
Ans. 3 2. Neuraminidase
ovarian primaries are indolent tumours 3. Glycoprotein
Robbin’s 8th edition Page no 1049 4. Protein matrix (NS 1)
More commonly of placental origin, the choriocarcinoma,
like the endodermal sinus tumor, is an example of extra- Ans. 2
embryonic differentiation of malignant germ cells. It is  Myxovirus (Influenza)
generally held that a germ cell origin can be confirmed  Elution (d/t neuraminidase)– only in Myxovirus
only in the prepubertal girl, because after this age an origin (except RSV & Measles)
from an ovarian ectopic pregnancy cannot be excluded.
Most ovarian choriocarcinomas exist in combination with 20. A newborn presents with excessive frothing. CXR
other germ cell tumors, and pure choriocarcinomas are given below
extremely rare. They are histologically identical to the
more common placental lesions, described later. The
ovarian primaries are aggressive tumors that generally
have metastasized widely through the bloodstream to the
lungs, liver, bone, and other viscera by the time of
diagnosis. Like all choriocarcinomas they elaborate high
levels of chorionic gonadotropins, which is sometimes
helpful in establishing the diagnosis or detecting
recurrences. In contrast to choriocarcinomas arising in
placental tissue, those arising in the ovary are generally
unresponsive to chemotherapy and are often fatal.

18. The following are variants of papillary carcinoma Which of the following statements are not correct?
of thyroid except 1. Polyhydramnios
1. Tall cell variant 2. Absent stomach bubble
2. Diffuse sclerosing variant 3. Type C is commonest
3. Follicular variant 4. Associated with dextrocardia
4. Anaplastic variant
Ans. 4. Associated with dextrocardia
Ans. 4
Anaplastic variant 21. Tumor that follows rule of 10 is -
Robbin’s 8th edition Page no 1122 1. Pheochromocytoma
Papillary carcinomas are the most common form of 2. Oncocytoma
thyroid cancer. They occur throughout life but most often 3. Lymphoma
between the ages of 25 and 50, and account for the 4. Renal cell carcinoma
majority of thyroid carcinomas associated with previous
exposure to ionizing radiation. The most common variant, Ans. is ‘1’ i.e., Pheochromocytoma
and the one most liable to misdiagnosis, is the follicular o Pheochromocytomas usually subscribe to a convenient
variant, which has the characteristic nuclei of papillary "rule of 10s":-
carcinoma but has an almost totally follicular architecture.  10% of pheochromocytomas arise in association
A tall-cell variant is marked by tall columnar cells with with one of several familial syndromes. These
intensely eosinophilic cytoplasm lining the papillary include the MEN-2 A & MEN-2B syndromes, type I
structures. An unusual diffuse sclerosing variant of neurofibromatosis, von Hippel-Lindau syndrome
papillary carcinoma occurs in younger individuals, & Sturge-Weber syndrome
including children. The tumor demonstrates a prominent  10% of pheochromocytomas are extra-adrenal,
papillary growth pattern, intermixed with solid areas occurring in sites such as organ of Zuckerkandl &
containing nests of squamous metaplasia. There is carotid body, where these chromaffin-negative
5
tumors arc usually called paragangliomas to 3. Adenyl cyclase
distinguish them frompheochromocytomas. 4. Guanyl cyclase
 10% of nonfamilial adrenal pheochromocytomas
are bilateral; this figure may rise to 70% in cases Ans. is ‘2’ i.e., Phosphodiesterase – 5
that are associated with familial syndromes. o Sildenafil is used to treat Erectile Dysfunction
 10% of nonfamilial adrenal pheochromocytomas o Mechanism of action:
are bilateral: this figure may rise to 70% in cases
that areassociated with familial syndromes, Nitic Oxide(NO)
 10% of adrenal pheochromocytomas arc Activation of Guanylyl PDE- 5  Sildenafi
biologically malignant, although the associated cyclase ↓
hypertension represents a serious & potentially ↓
lethal complication of even "benign" tumors, GTP cGMP → Degraded product
 10% of adrenal pheochromocytomas arise in Reduction of intracellular
childhood, usually the familial subtypes, and with calcium
a strong malepreponderance. The nonfamilial ↓
pheochromocytomas most often occur in adults Smooth muscle relaxation
between 40 & 60 years of age, with a slight female
preponderance. o Penile tumescence during sexual arousal is improved,
but it has no effect on penile tumescence in the
22. The following are false of thymomas except absence of sexual activity, because sexual stimulation
1. Equal sex incidence results in smooth muscle relaxation of the copus
2. Neoplastic cells are thymic epithelial cells and cavernosum, increasing the inflow of blood.
thymocytes o Sildenafil in addition, weakly inhibits the isoenzyme
3. They are common in posterior mediastinum PDE-6 which is involved in photoreceptor
4. Tumors that have a substantial proportion of transduction of retina  impairment in colour vision
medullary-type epithelial cells are usually invasive. especially blue green discrimination

Ans. 1 24. Not true about Wilson’s disease


Equal sex incidence 1. Defect in ATP7B
Robbins 8th edition Page no 636 2. Serum ceruloplasmin is increased
The designation “thymoma” is restricted to tumors of 3. Kayser Feischer ring pathognomonic
thymic epithelial cells. Such tumors typically also contain 4. Penicillamine is used in treatment
benign immature T cells (thymocytes). The tumors usually
occur in adults older than 40 years of age; thymomas are Ans. 2. Serum ceruloplasmin is increased
rare in children. Males and females are affected equally. Wilson’s disease
Most arise in the anterior superior mediastinum, but Autosomal recessive; Liver symptoms first, neurologic
sometimes they occur in the neck, thyroid, pulmonary symptoms later
hilus, or elsewhere. They are uncommon in the posterior Dystonia, tremors, basal ganglia problems
mediastinum. Macroscopically, thymomas are lobulated, Kayser-Fleischer rings-pathognomonic
firm, gray-white masses of up to 15 to 20 cm in size. They MRI shows lesions in thalamus and basal ganglia
sometimes have areas of cystic necrosis and calcification. Best screen-serum ceruloplasmin (decreased)
Most are encapsulated, but 20% to 25% of the tumors Increased urinary Cu excretion
penetrate the capsule and infiltrate perithymic tissues and Confirm with liver biopsy-increased Cu content
structures. Noninvasive thymomas are most often Treatment
composed of medullary-type epithelial cells or a mixture of - Chelation with penicillamine (slows progression)
medullary- and cortical-type epithelial cells. The - Definitive treatment with liver transplant
medullary-type epithelial cells are elongated or spindle-
shaped. There is usually a sparse infiltrate of thymocytes, 25. A ex-preterm infant who has lower limb weakness
which often recapitulate the phenotype of medullary has the following picture in MRI. What does the
thymocytes. In mixed thymomas there is an admixture of neonate have?
polygonal cortical-type epithelial cells and a denser
infiltrate of thymocytes. The medullary and mixed patterns
together account for about 50% of all thymomas. Tumors
that have a substantial proportion of medullary-type
epithelial cells are usually noninvasive.

23. Sildenafil acts by inhibiting -


1. Phosphodiesterase - 2
2. Phosphodiesterase - 5
6
B. Inspiratory reserve volume-2 L
C. Expiratory reserve volume-1 L
D. Air left in the lungs after a maximal expiratory effort is
the residual volume-1.3L. This is 20% of total lung
capacity

28. "Menke's disease" is a disease of -


1. Impaired zinc transport
2. Impaired copper transport
3. Impaired magnesium transport
1. Periventricular leukomalacia 4. Impaired molybdenum transport
2. Canavan disease
3. Duchene’s muscular dystrophy Ans. is '2' i.e., Impaired copper transport
4. Neuronal ceroidlipofuscinosis Menke's disease is caused due to defect in the copper
transport.
Ans. 1. Periventricular leukomalacia  There is defect in the transport of copper present in
PVL the intestinal mucosa to the blood stream.
PVL refers to injury of the cerebral white matter, generally  The mucosal lining of intestine contains high level of
more severe in the deep that in superficial white matter, copper bound to metallothionein protein.
and exists in 2 major forms:  Rather than being transporated to bloodstream, the
 cystic copper remained in the mucosa and was lost when
 diffuse Typically cystic PVL (cPVL) is located deep in intestinal cells were naturally soughed off.
the cerebral white matter (WM), usually dorsal and Menkes disease is caused due to defect in the "MNK"
lateral to the external angles of the lateral ventricles). gene.
(ref: Volpe JJ. The encephalopathy of prematurity — brain  The protein normally Junction by moving copper from
injury and impaired brain development inextricably the intestinal mucosal cells into the blood stream,
intertwined. SeminPediatr Neurol. 2009 Dec; 16(4): 167– where
78.) it is bound by proteins such as albumin and
transported to organs and tissues.
26. Which cell serves as progenitor cell following lung Serum copper is critical for the functioning of several
injury for repair? enzymes
1. Merkel cells  Lysyl oxidase → is important for the cross linking of
2. Airway epithelial cells collagen and elastin such that deficiencies lead to
3. Clara cells problems in connective tissues such as bones
4. Neuroepithelial bodies  Cytochrome oxidase → Involved in temperature
maintenance
Ans. 3  Tyrosinase → Necessary for pigmentation
Ganong review of medical Physiology 24th ed p.621 Clinical features of menkes disease
Conducting airways  Growth retardation
A. First 16 generations starting from trachea  Coarse hair, brittle and ivory white (result of
B. Ciliated- “Mucociliary escalator”. beat at rates of 10–15 depigmentations). The hair fibres are twiisted and
Hz. Moves 16 mm/min broken helically (kinky hair),
C. Basal cells-progenitor cells following injury is present  Seizures
D. Clara cells- nonciliated, secrete defense markers, serve  Cerebral and cerebellar degeneration (postmortem
as progenitor cells after injury analysis)
E. Epithelial cells-lung defense- IgA, collectins,defensins  Hypothermia
 Thrombosis
27. Value of residual volume in lung is  Poor bone development
1. 1000 ml  Increased tendency towards aneurysms
2. 2000 ml
3. 3000 ml 29. The Diagnosis of this disorder
4. 1300 ml

Ans. 4
Ganong review of medical Physiology 24th ed p.629
Lung volumes
A. Amount of air that moves into the lungs with each
inspiration-Tidal volume(500-750 ml)

7
5 days

31. Contraction stress test false is -


1. Oxytocin not used
2. Invasive method
3. Detects fetal well being
4. Negative test is associated with good fetal outcome

Ans. is '1' i.e., Oxytocin not used


Contraction stress test (CST) (syn : Oxytocin challenge
test)
 It is an invasive test to asses fetal respiratory well
being during pregnancy.
 It detects alteration in FHR in response to uterine
1. Normal X ray contraction induced by oxytocin indicating hypoxia,
2. Duodenal atresia  Interpretations
3. Jejunal Atresia  Negative test indicated good outcome
4. Malrotation  Positive test is associated with increased incidence of
IUD, fetal distress & low Apgar score,
Ans: 2.  There is 50% chance of false positive results.
Double bubble- Duodenal atresia
Triple bubble- jejunal atresia 32. Maximum chance of transmission during delivery-
1. HSV
2. CMV
30. Emergency contraceptive should must be started 3. VZV
with in how much time after unprotected intercourse- 4. Rubella
1. 24 hrs
2. 48 hrs Ans. is ‘1’ i.e., HSV
3. 72 hrs "Transplacental infection by HSV is not usual. The
4. 96 hrs fetus becomes affected by virus shed from the cervix
or lower genital tract during vaginal delivery."
Ans. is ‘3’ i.e., 72 Hrs.  Overall most common congenital infection is CMV.
Emergency contraception (post coital contraception)  But most common congenital infection during delivery
Indications is HSV.
o Unprotected intercourse
o Condom rupture 33. Drug causing abruption placenta –
o Missed pill 1. Methadone
o Delay in taking POP for more than 3 hours 2. Cocaine
o Sexual assault or rape & first time intercourse. 3. Amphetamine
"Morning after pill: ethinyl-estrediol 2.5 mg, premarin 15 4. Fluoxetine
mg, the drug is taken orally twice daily for 5 days.
Beginning soon after exposure but not later than 72 hrs.” Ans. is ‘2’ i.e., Cocaine
Etiology of abruptio placenta
Post-coital contraceptives Risk factors
Drug Dose Pregnancy  Multigravida
rate(%)  Advancing age of mother
Levonorgestrel 75 mg stat & after 0-1  Poor socioeconomic status
12 hrs  Malnutrition
Ethynyl estradiol 2 tab stat & 2 0-2  Smoking
50 mcg+ days after  Hypertension
norgestrel 0.25 Causes
mg  Trauma
Conjugated 15 mg BD for 5 0-0.6  Sudden uterine decompression eg:- following delivery
estrogen days of the 1st baby of twins, sudden escape of liquor amnii
Ethinyl estradiol 2.5 mg BD for 5 0-0.6 in hydramnios, PROM,
days  Short cord
Mifepristone 10 mg single dose 0-0.6  Supine hypertension syndrome
Copper IUDs Insertion within 0-0.1  Placental anomalies
8
 1 Sick placenta  Patients who present late in the course of appendices
 Folic acid deficiency with a palpable or radiographic ally documented
 Uterine factors like septate uterus or a submucous mass(abscess or phlegmon) are treated with- (abscess
fibroid or phlegmon) are treated with-
 Torsion of uterus  Conservative therapy and interval appendectomy 6 to
 Cocaine abuse 10 weeks later.(conservative management includes
 Thrombophillias intravenous antibiotics and fluids as well as bowel
 Prior abortion rest.)
 Patients with large abscesses, greater than 4 to 6 cm in
34. Peritonitis in acute appendicitis is caused by - size, and especially those patients with abscess and
1. Early rupture of appendix high fever, benefit from abscess drainage.
2. Late rupture of appendix
3. Fecolith 35. Champagne bottle appearance is seen in -
4. Old age 1. Varicose veins
2. DVT
Ans. is ‘1’ i.e., Early rupture of appendix 3. Lipodermatosclerosis
 Perforation of appendix in acute appendicitis 4. Venous ulceration
commonly leads to an abscess cavity walled off by the
small bowel loops and the omentum, forming a Ans. is ‘3’ i.e., Lipodermatosclerosis
phlegmon. Rarely the appendix may perforate freely  Lipodermatosclerosis refers to a skin change of lower
into the peritoneal cavity and cause generalized legs that often occurs in patients who have chronic
peritonitis. This usually occurs in cases of early venous insuficiency. This is a type of panniculitis
rupture of the appendix as inflammatory process did (inflammation of subcutaneous fat),
not get time to be localized by the omentum and bowel  Inverted champagn bottle or bowling pin appearence
loops. is seen,
 Usually the rupture of appendix is a late sequelae of  This is also seen in "Charcot-Marie-Tooth disease
appendicitis, usually occurring 48 to 72 hours from
the onset of symptoms. 36. Bismuth classification in which class, hepatic
Also know: duct confluence is involved -
 The site of rupture is commonly just beyond the point 1. I
of obstruction, along the antimcsenteric border of 2. II
appendix. 3. III
 Fecaliths are the most common cause of appendiceal 4. IV
obstruction.
 Lets also see the management of appendicitis: Ans. is ‘2’ i.e., Type II
Management of appendicitis: Bismuth - Corlette classification is used to classify
 The treatment of appendicitis is appendectomy. (It can cholangiocarcinoma.
be done open or laparoscopically) Type I - Common hepatic duct involvement,
 Prophylactic antibiotics are indicated preoperatively. Type II - CHD + bifurcation/confluence of hepatic ducts,
Postoperative antibiotic coverage is of no use in Type III - a) Extension to right secondary intrahepatic duct.
simple (uncomplicated) acute appendicitis. If b) Extension to left secondary intrahepatic duct.
perforated or gangrenous appendicitis is found, Type IV - Involving both right and left secondary
antibiotics are continued until the patient is afebrile intrahepatic ducts.
and has a normal white blood cell count.
Perforated Appendicitis 37. Parathyroid glands are removed by surgery, for
recurrence investigation of choice-
 Rupture is suspected in the presence of fever with a
temperature of >39°C (102°F) and a white blood cell 1. Technetium scan
2. SPECT
count of >18,000cells/mm3.
3. MRI
 The management of perforation depends on the
4. Neck ultrasound
nature of the perforation.
 If the perforation is free causing intraperitoneal
Ans. is '1' i.e., Technetium scan
dissemination of pus and fecal material, urgent
 Preoperative localisation tests for parathyroid glands
laparotomy is done for appendectomy and irrigation
are –
and drainage of the peritoneal cavity,
1. Tc 99m labeled sestamibi scan (>80% sensitive)
 If the perforation is contained it would result in an
2. Single positron emission computed tomography
appediceal mass or abscess. This is managed as
(SPECT).
descnbed under.
3. CT and MRI scan.

9
4. Neck ultrasound. receptor and not the other have an intermediate
response rate.
38. True about her2/neu overexpression in Carcinoma  The HER2 receptor (previously called HER2/neu, or
breast - ERBB-2 receptor) belongs to the epidermal growth
1. Responds well to taxanes factor receptor (EGFR) family of receptors, which are
2. Good prognosis critical in the activation of subcellular signal
3. Responds well to monoclonal antibodies transduction pathways controlling epithelial cell
4. Seen only in breast cancer growth and differentiation and possibly angiogenesis.
 Amplification of HER2 or overexpression of its protein
Ans. is ‘3’ i.e., Responds well to monoclonal antibodies product is observed in 18 to 20 percent of human
Prognostic markers of breast cancer breast cancers.
A. Indices of proliferation (PCSA, Ki-67)  HER2 overexpression is also noted in other tumors
 PCNA & Ki-67 expression are poor prognostic markers such as esophagogastric tumors, lung, ovary & head
 PCNA (proliferating cell nuclear antigen) and Ki-67 and neck squamous cell ea. (In all of these sites, HER2
expression arc positively correlated with p53 overexpression has been identified as a negative
overexpression, high s-phase fraction, aneuploidy. prognostic factor.)
high mitotic index, and high histologic grade in human Following points arc to be noted about HKR2
breast cancer specimens, and arc negatively correlated overexpression in breast ca;
with estrogen receptor content.  Prognostic value of H ER2 — HER2 overexpression is a
B. Indices of Apoptosls (bcl-2 and box: bell ratio) poor prognostic marker. HER2 overexpression is
 Overexpression of bcl-2 and reduced box : bc12 ratio associatedwith high rates of disease recurrence and
are poor prognostic markers death in the absence of adjuvant systemic therapy.
 Bcl-2 is the only oncogene that acts by inhibiting  Predictive value of HER2 — MLR2 status predicts
apoptosis rather than by directly increasing cellular response to specific therapies:
proliferation. 9 Rax is death-signal protein and inhibits  Patients with high levels of HER2 expression benefit
the anti-apoptotic actions of bcl-2. from treatment with agents that target HER2, such as
 Overexprcssion of bcl-2 and a decrease in the bax : bcl- trustuzumab (a monoclonal antibody) and lapatinib.
2 ratio correlates with high histologic grade, the  HER2 status appears to predict resistance or
presence of axillary lymph node metastases, and sensitivity to different types of chemotherapeutic
reduced disease-free and survival rates. agents, including anthracyclines and taxanes.
C. Indices of angiogenesis (VEGF A angiogenesis index)  Women whose rumors overexpress HER2 appear to
 Angiogenesis is necessary for the growth and derive greater benefit from anthracycline-based
invasiveness of cancer and promotes cancer adjuvant therapy than from adjuvant therapy that is
progression through several different mechanisms. alkylating agent-based, such as CMF
VEGF stimulates the growth of new blood vessels. (cyclophosphamide,methotrexate, fluorouracin.
 Overexpression of VEGF (vascular endothelial growth  Relationship between HER2 overexpression and
factor) is poor prognostic marker. taxanes is still under study with various studies giving
 Bevacizumab is a monoclonal antibody that blocks conflicting reports.
vascular endothelial growth factor. Bevaci/umab is the  HER-2 positivity is associated with resistance to
first clinically available angiogenisis inhibitor and is endocrine therapies.
used for metastatic breast cancer (Anliangiogenesis
breast cancer therapy) 39. True about achalasia cardia is -
D. Growth factor receptors 1. Dysphagia is a presenting symptom
 HER 2/neu o: EGFr overexpression are poor 2. The cause is the absence of Auerbach'splexes
prognostic markers. 3. Esophagectomy is the treatment
 Patients whose tumors ovcrexprcss HER-2/ncu are 4. Motilily-improving agents are used in treatment
given anti-l IER-2'neu therapy - Trastuzumab
(Hcrceptin). It is a recombinant humanized Ans. ‘1’ i.e., dysphagia is a presenting symptom
monoclonal antibody directed against HER-2/neu. Achalasia
E. p53  Achalasia cardia is a motor disorder of the esophageal
 p53 overexpression is poor prognostic marker smooth muscle in which the LES does not relax
F. Steroid hormone receptors (ER, PR) normally with swallowing, & the esophageal body
 Patients with hormone receptor positive tumor show undergoes nonperistaltic contractions. (Primary
good response to hormone therapy. peristalsis is absent or reduced).
 Tumors positive for estrogen or progesterone  The pathogenesis of achalasia is poorly understood.
receptors have a higher response rate to endocrine It involves:
therapy than tumors that do not express estrogen or  Neurogenic degeneration either idiopathic or due to
progesterone receptors. Tumors positive for one infection. The degenerative changes are either
10
intrinsic (degeneration of ganglion cells of Aurebach’s transmitted inhibitory effect of CCK is absent owing to
myentric plexus) or extrinsic (extraesophageal vagus the loss of inhibitory neurons.
nerve or the dorsal motor mucleus of vagus). Also know
 Pharmacologic studies suggest dysfunction of  Schwartz writes-"Although complete absence of
inhibitory neurons containing nitric oxide and peristalsis in the esophageal body has been proposed
vasoactive as the major abnormality, present evidence indicates
intestinal polypeptide in the distal esophagus (LBS). achlasia is a primary disorder of the LES”. The LES fails
The clolinergic innervation of the LES is intact or to relax on deglutition, elevation of intraluminal
affected only in advanced disease, esophageal pressure esophageal dilatation subsequent
 As a result of the abnormality, the LES fails to relax, loss of primary peristalsis in the body of esophagus,
primary peristalsis is absent in esophagus which  Other options are described ahead,
dilates. As  Over 20 year period a patient will have upto 80% risk
the disease progresses the esophagus becomes of esophageal CA. Sq. cell CA is the most frequent type
massively dilated and tortuous. seen
Clinical findings
 Both sexes arc equally affected 40. A new born with respiratory distress from day 1
 May develop at any age but peak years are from 30 to has this X-ray. Identify
60.
 Classical clinical symptom is progressive dysphagia for
both solids and liquids. Dysphagia is worsened by
emotional stress and hurried eating,
 Regurgitation and Pulmonary aspiration occur
because of retention of large volumes of saliva and
ingested food in the esophagus.
 Esophagitis with ulceration may occur with chronic
retention of food.
 Pain is infrequent in classical achlasia but a variant
called vigrous achlasia is characterized by chest pain
and esophageal spasms that generate non-propuloine
high-pressure waves in the body of the esophagus.
Diagnosis
 Chest x-ray - shows absence of gastric air bubble, an 1. CCAM
air-fluid level in the mediastinum in the upright 2. Congenital lobar emphysema
position representing retained food in oesophagus, 3. Pneumothorax
 Barium swallow - Shows dilated esophagus with 4. CDH
tapering narrowing in the terminal end of esophagus
describe as 'birds beak1 appearance. Fluoroscopy Ans. 2. Congenital lobar emphysema
shows loss of normal peristalsis in the lower two Congenital lobar emphysema
thirds of esophagus. The most common site of involvement for congenital lobar
 Endoscopy may be done to rule out any secondary emphysema is
cause of achlasia eg. carcinoma, stricture at LES. 1. Left upper lobe (40% to 50%)
Manometry 2. Right middle lobe (30% to 40%)
 It's the most vonfrmatory investigation. 3. Right upper lobe (20%)
 It is able to distinguish between various forms of 4. Lower lobes (1 %), and multiple sites for the
motor disorders of esophagus remainder.
 Manometric characteristics of Achlasia 5:1 male predominace
‐ Incomplete lower esophageal sphincter relaxation (<
75% relaxation) Chest x ray findings
‐ Elevated LES pressure Lobar inflation
‐ LOSS of primary peristaltic waves in the esophageal Mediastinal shift
body, hut disorganized muscular activity may he Atelectasis of adjacent lungs
present. Ipsilateral hemidiaphram flattening
‐ Increased intraesophageal baseline pressure relative Mimic tension pneumothorax
to gastric baseline. D/D:
 Cholecystokinin (CCK) which normally causes a fall in Pneumothorax
the sphincter pressure, paradoxically causes Pneumatocoele
contraction of the LES (the CCK test). This paradoxical Giant CCAM
response occurs because, in achlasia the neurally
41. Most common type of intussusceptions-
11
1. Multiple 2. It is a spinal reflex
2. Colocolic 3. It is usually present at birth
3. Honleal 4. Its disappearance is essential for the child to be able to
4. Ileocolic turn over

Ans. is '4' i.e., Ileocolic Ans. 4. Its disappearance is essential for the child to be
Types of intussusception in decreasing order able to turn over
 Ileocolic (-77%) ATNR (Asymmetric tonic neck reflex is a brain stem
 Ileoileo-colic(~12) reflex)
 Ueoileai(~5%) How to elicit?
 Colocolic(-2%)  Baby to be lying supine, head in mild line, awake or
 Multiple (1%) light sleep.
 Retrograde  Shoulder horizontal
Intussusception  Head turned to right till jaw over right shoulder
 Intussusception is the telescoping of one pon  Held for 15 sec and released.
childhood (3 months to 6 years)
 It is the most common cause of intestinal obstruction
ear
 An intussusception is composed of three parts.
‐ The entering or inner tube - intussusception
‐ The returning or middle tube
‐ The sheath or outer tube - intussuscipiens.

Clinical features
 Intussusception classically produces severe cramping
abdominal pain in an otherwise healthy child. The
child often draws his or her legs up during the pain
episodes and is usually quite during the intervening
periods.
 Vomiting is almost universal Normal response:
 Initially the passage of stools may be normal while  Passive reflex elicited
later on blood mixed with mucus is evacuated – red  Mental extension, occipital flexion
currant jelly stool  Arm and leg on the mental side extend
 An abdominal mass may be plapated – a sausage  Arm and leg on the occipital side flexes.
shaped mass, which may increase in size and firmness  If it occurs spontaneous, it is an active reflex.
during the paroxysms of pain.
 It I usually present from 1 month age
 There may be an associated feeling of emptiness in the Abnormal response:
right fossa(Sign of Dance)
 Sustained or exaggerated response –
 On rectal examination, blood-stained mucus may be
 Consistent failure to move an extremity
found on the finger. Occasionally in extensive ileocolic
 Persistence of this reflex beyond 6 months.
or colocolic intussusception, the apex may be palpable
 Obligatory ATNR
or even protrude from the anus.
Clinical implication:
 Diagnosis can be made on:
 In severe spastic cerebral palsy this reflex persists and
 Barium enema which shows characteristic
may increase.
‐ Claw sign
 This persistence of the reflex prevents child rolling
‐ Coiled spring sign
from prone to supine or vice versa.
 On ultrasound which shows
(M Singh, Clinical methods in Pediatrics)
‐ Target sign
‐ Pseudokidney sign
43. A 1-month-old fair-haired, fair-skinned baby
 When the clinical index of suspicion is high, presents with projectile vomiting of 4 days' duration.
hydrostatic reduction by contrast agent or air enema
Physical exam reveals a baby with eczema and a musty
is the diagnostic and the therapeutic procedure of
odour. Whichscreening test would most likely be
choice. Hydrostatic reduction is contraindicated in
abnormal?
peritonitis or hemodynamic instability.
1. Ferric chloride test
2. Benedict test
42. Which of the following is true regarding
3. Rothera Test
Asymmetric tonic neck reflex?
4. Diazo test
1. Persistence beyond 6 months is normal
12
Erythema nodosum
Ans. 1. Ferric chloride test • Erythema nodosum in itself is not a disease
Phenyl Ketonuria • Associated with many infections (e.g., group A
• Defect in Phenylalanine hydroxylase streptococcal infection, tuberculosis, Yersinia
• Mental retardation, vomiting, growth retardation, infection,histoplasmosis, coccidioidomycosis)
hyperactive, hepatomegaly, vomiting, purposeless • Inflammatory states (e.g., inflammatory bowel disease,
movements, athetosis, seizures systemic lupus erythematosus, juvenile rheumatoid
• Exam: Fair hair, fair skin, blue eyes, rash, tooth arthritis, sarcoidosis)
abnormalities, microcephaly • Administration of drugs (e.g., sulfonamides, phenytoin,
• Normal at birth , progressive MR oral contraceptives)
Treatment
• Cofactor tetrahydrobiopterin(BH4) administration can 46. A newborn being evaluated for hyperammonemia
reduce phenylalanine levels has elevated orotic acid but decreased/absent
• Abnormal urinary metabolites are detected by citrulline. What is the most likely diagnosis?
Guthrie or ferric chloride test 1. Citrillinemia
• Mental retardation can be prevented by restriction of 2. Arginosuccinic aciduria
phenylalanine in the diet right from birth which 3. OTC deficiency
should continue for 8-10 year 4. CPS deficiency
Urine Odours in IEMs
Musty-PKU Ans. 3. OTC deficiency
Cabbage-Tyrosinemia Urea Cycle
Maple syrup-MSUD
Sweaty feet-Isovaleric acidemia, glutaric acidemia type II
Cat urine-3-methylcrotonylCoA carboxylase and multiple
carboxylase deficiencies

44. Which of the following is not a feature of erb’s


palsy?
1. Inability to abduct shoulder
2. Inability to externally rotate shoulder
3. Loss of grasp
4. Inability to supinate forearm

Ans. 3. Loss of grasp


Erb’s palsy

Flow chart differentiating Hyperammonemia

Police man receiving tip position – C5, C6 root injury


Klumpke’s – C7, C8, T1 injury

45. Erythema nodosum is associated with all of the


following except
1. Sarcoidosis
2. Yersinia infection
3. Pseudotumour cerebri
4. Inflammatory bowel disease

Ans. 3. Pseudotumour cerebri

13
Neonatal hyperammonemia

Symptoms in first 24hr life Symptoms after 24hr life

Premature Full term Acidosis No acidosis

THAN IEM OA or PC deficiency Organic Urea cycle


acidemias defects

Plasma AA

Absent citrulline Cit.moderately Cit. highly Elev;


Elev; ASA No ASA present
present

Urine Orotic acid Argininosuccinic Citrullinemia


aciduria

Low Elev

CPS deficiency OTC deficiency

Urea cycle defects basis. He is diabetic and takes an oral hypoglycemic.


Urea cycle defect: after the introduction of feeds and no his blood sugars have been slightly higher than
acidosis normal. Exam reveals a tender RUQ, positive Murphy's
Mx – Nil oral. Sodium Benzoate and arginine, Peritoneal sign
dialysis
Autosomal recessive , risk in each pregnancy is 25 %

47. Which one of the following is not considered as


firstrank symptom of schizophrenia?
1. Auditory hallucinations
2. Insertion of thoughts
3. Delusional perceptions
4. Compulsive acts that relieve the tension

Ans. Is '4' i.e., Compulsive acts that relieve the tension


Schneider's first rank symptoms
1. Hallucinations:- Audible thoughts, voices
commenting on one's action, voices heard arguing two
or more hallucinatory voices discussing the subject in 1. Gas in gall bladder wall
third person (third person hallucination). 2. Gall stone ileus
2. Thought attention phenomena:- Thought insertion, 3. Aortic aneurysm
thought withdrawal, thought broadcasting. 4. Liver abscess
3. Passivity phenomena:- Made feeling (affect), made
impulse, mode volition (acts), somatic passivity. Ans. 1. Gas in gall bladder wall
4. Delusional perceptions (Primary delusions).
49. Max duration of time spent is in NREM stage –
48. A 55 year old male presents to the ER with 1. I
complaints of RUQ pain, fever and chills. He says he 2. II
has not been feeling well for the past 3 days. He had 3. III
similar pain 3 months ago but it subsided with some 4. IV
zantac. he denies any trauma and drinks on a social
14
Ans. is ‘2' i.e., II 2. Alteration in menial function
 REM sleep ocupies 20-30% of total steep and NREM 3. VIth nerve palsy
sleep ocupies 60- 70% (state I: 5-10%, stage II: 40- 4. Ataxia
50% stage III & IV: 15-20%).
Stages of Sleep Ans. is ‘1' i.e., Cog-wheel rigidity
o Sleep is basically divided into two phases : -  Wernicke's encephalopathy is characterized by a triad
A) Non-REM (NREM) or slow-wave sleep: - It is called of global confusion (alteration in mental
Non-REM sleep because no rapid eye movement (REM) is function),Ophthalmoplegia (Bilateral 6th nerve palsy)
there on eletrooculogram, rather there is slow or quiscent and ataxia.
eye movement. It is also referred as S-sleep (synchronized Wernicke's encephalopathy
sleep) or quiet sleep or orthodox sleep because there  It is an acute neurological disorder characterized by a
diminished physiological functions and a low level of classical triad of ataxia, ophthalmoplegia and
overall activity. It is further divided into: - confusion. The clinical findings include : -
1. Stage 1: - It is a transition from wakefulness to sleep 1. Ocular: - Nystagmus, hi lateral lateral rectus palsy,
and is characterized by disappearance of alpha activity conjugate gaze palsies. The most common ocular
and appearance of theta activity. There is slow eye abnormality is nystagmus, not complete
movements. ophthalmoplegia. Less frequently noted are pupillary
2. Stage 2: - It is characterized by typical EEG activity i.e.. abnormalities such as sluggishly reactive pupil, ptosis,
Sleep spindles and 'K-complex'. No eye movement scotomas, and anisocoria.
occurs. 2. CNS (encephalopathy): - Global confusion, disinterest,
3. Stage 3 & 4: - Stage 3 & 4 are stages of deep sleep. inattentiveness or agitation. Rarely stupor and coma
These are also called slow wave sleep as these stages mayoccur.
are characterized by slow delta waves. Delta wave 3. Gait ataxia
appearance starts in stage 3 and dominated in stage 4. 4. Peripheral polyneuropathy: - Weakness, foot drop,
B) REM sleep: - The REM sleep is so named as there is decreased proprioception.
rapid roving movements of eye, i.e., rapid eye movement 5. Other symptoms: - Autonomic dysfunction
(REM)on electro-oculogram. It is also called paradoxical (tachycardia & postural hypotension). If there is
sleep because there is a paradoxical elevation of brain coexisting cardiovascular beriberi; hypothermia;
activity & metabolism and physiological activity. The high- abdominal symptoms (nausea, vomiting, abdominal
amplitude slow waves seen in stage 3,4 of REM sleep pain).
is replaced by rapid low voltage activity (beta wave).
Features of REM sleep are: - 51. A 40 year old female developed persistent oral
i. EEG  Beta wave, Reappearance of alpha wave, ulcers followed by multiple flaccid bullae on trunk and
Saw-tooth wave (low voltage fast activity), Ponto- extremities. Direct examination of a skin biopsy
genitai-occipital spikes. immunofluorescence showed intercellular IgG
ii. Dreaming deposits in the epidermis and suprabasal split with
iii. Other features: - Generalized muscular atony, acantholytic cells. The probable diagnosis is:
Penile erection, autonomic hyperactivity 1. Pemphigus vulgaris
(Increased pulse rate & BP) and intermittent 2. Pemphigoid.
movement of small muscle groups. 3. Erythema multiforme.
 The NREM (with its four stages) and REM sleep repeat 4. Dermatitis hepetiformis
several times a night in cyclic manner, 4-6 times
depending on the length of sleep. A typical cycle starts Ans. 1. Pemphigus vulgaris
with stage I of the NR£M sleep which is followed by Reference
second, third and fourth stages. After this the sleep Pemphigus Vulgaris
returns to stage 3 and 2. The first REM sleep occurs 1. It is an Auto Immune Mediated blistering skin
now, 70-90 minutes after the the commencement of diseasefother immune mediated blistering skin
sleep (Note: - Stage 1 NREM sleep occurs only once at
the start of the sleep as this is only a transition phase
between wakefulness and sleep. Once the sleep starts diseases are Pemphigus Foliaceus, Bullous
the cycle rotates between stage 2,3, 4 NREM and REM Pemphigoid, Cicatrical Pemphigoid, Dermatitis
sleep). REM sleep occupies 20-30% of total sleep and Herpetiformis, Epidermolysis Bullosa Acquista and
NREM sleep occupies 60-70% (Stage-1 -> 5-10%; Linera IgA disease)
Stage 2 -> 40-50%; Stage 3 & 4 -H5-20%). 2. HLA-DR4 and DRW6
3. Acantholyssis and Intraepidermal L\Blisters are
50. All are associated with Wernicke's encephalopathy haracteristic features
except - 4. Acantholytic cells are Round cells with
1. Cog wheel rigidity hyperchromatic nucleus and Perinuclear Halo

15
5. Nikolsky's Sign is seenfOther diseases with Nikolsky's Pemphigus is seen after administration of
Sign are Pemphigus, Pophyria, Steven Johnson's Toxic Penicillamine and Captopril
Epidermal Necrolysis, Staphylococcal Scalded Skin • Pemphigus Vegitans is the least common
Syndrome) Nikolsky's Sign is negative in Pemphigoid
6. Tzanck's Test is used. Let us see the differences between Pemphigus and
7. Tzanck's Cell is Keratinocyte Phempigoid
8. IgG antibodies are seen
9. Row of Tombstones appearance is present in Features Pemphigus Pemphigoid
PemphigusVulgaris Age 40-60 60-80
HPE-Row of Present Absent
Explanation Tombstones Present Absent
1. Pemphigus vulgaris presents with Acantholysis and Nikolsky's Sign Intraepidermal Sub Epidermal
Intraepidermal Bullae. Bulla Location Flacced Tense
2. Pemphigoid presents with subepidermal bullae IgG Bulla Features Involved Not involved
Antobodies are present against Basement Membrane Mucosa (eg Oral) Present Absent
and usually affect the elderly in 60 - SO years and Acantholysis
Mucosal Involvement is rare. There is no statistical
association with internal malignancy and HLA and 52. MRI in a patient demonstrates periventricular
Direct Immunonoflourescence reveals IgG and C3 high–signal intensity lesions. What is your diagnosis
deposits.
3. Erythema multiforme is usually due to Viral (Herpes)
or other Bacteria and Fungus. Lesions come in crops
and last for 2 ot3 weeks, and commonly affect the face,
dorsal surface of hand and feet and ecxtensor sirface
of forearm and legs, palms and soles. Target Lesions
(Bullet's Eye lesions) are specific and they consist of 3
conventric zones of color changes and are mostly
found acrally on the hand and feet in Erythema
Multiformae Minor
4. Dermatitis herpetiformis presents as intensely
pruritic, chronic, papulovesicular lesions that are
symmetrically distributed over extensor surfaces.
Associate with gluten sensitive Enteropathy. More
than 90 % express HLA B8 HLA DRW3 and HLA
DQW2. Subepidermal Blisters with Neutrophils ar
seen and Granular IgA deposits are seen in the dermal
papillae. The IgA antibodies are against Gliadin.
Treatmetn is with Dapsone and if the patient cannot
tolerate Dapsone, Sulphapyridine is substituted. Diet 1. Multiple Sclerosis
should exclude Gluten which is present in Barley ,Rye 2. Toxoplasmosis
and Wheat. Oats are tolerated and Gluten is not seen 3. Periventricular leukomalacia
with Rice. 4. Hypertensive Encephalopathy

Tips Answer 1. Periventricular plaques, head MRI


• PemphiguS has Superficial (Intra epidermal) blisters This radiology finding is characteristic for multiple
• PemphigoiD has Deep (Sub Epidermal) blisters sclerosis. Neural inflammation increases the permeability
• And among Pemphigus of the blood brain barrier, which allows IV contrast
o P.Erythematosus and P.Foliaceous show material to accumulate in the periventricular region.
intraepidermal Acantholytic Split in Stratum
Granulosum (Pnemonic EFG)and 53. All are grave sign of GERD in child except?
o P.Vulgaris and P.Vegetans show Intraepidermal 1. Heartburn
Acantholytic split in between the Basal layer and the 2. Dysphagia
Prickle cell layer 3. Odynophagia
• The Nikolsky sign is positive in pemphigus and more 4. Jaundice
rarely toxic epidermal necrolysis.
• Pemphigus Vulgaris is due to autoimmunity directed Ans. 1. Heartburn
against Cadherin This is one of the most frequent, early and constant feature
• Pemphigus Foliaceus is due to autoimmunity directed of GERD.
against Desmoglein and this Drug induced
16
Once alarm factors are excluded, patients with typical International Staging System
GERD do not need further evaluation and are treated Stage Median Survival, Months
empirically. Upper endoscopy is indicated to exclude 2M < 3.5, alb 3.5 I (28%) 62
mucosal injury in cases with atypical symptoms, symptoms 2M < 3.5, alb < 3.5 II (39%) 44
unresponsive to acid suppressing drugs, or alarm factors. or 2M = 3.5-5.5
2M > 5.5 III 29
Odynophagia (33%)
Unexplained weight loss Note: (#), % patients presenting at each stage.
Recurrent vomiting Abbreviations: 2M, serum 2-microglobulin in mg/L; alb,
Occult or gross gastrointestinal bleeding serum albumin in g/Dl
Jaundice
Palpable mass or adenopathy Serum 2-microglobuliu is the single most powerful
Family history of gastrointestinal malignancy predictor of survival and can substitute for staging. 2-
Odynophagia Microglobulin is a protein of 11,000 mol wt with
Upper endoscopy is performed as the initial diagnostic test homologies to the constant region of immunoglobulin’s
in patients with unexplained dyspepsia who are >55 years that is the light chain of the class I major histocompatibility
old or who have alarm factors because of the elevated risks antigens (HLA-A, -B, -C) on the surface of every cell.
of malignancy and ulcer in these groups. The management Patients with 2-microglobulin levels <0.004 g/L have a
approach to patients <55 years old without alarm factors median survival of 43 months and thosewith levels >0.004
is dependent on the local prevalence of H, pylori infection. g/L only 12 months. Serum 2-microglobulin and albumin
levels are the basis for a three-stage International Staging
Alarm Symptoms that Indicate the Need for Upper System (ISS). It is also felt that once the diagnosis of
Endoscopy myeloma is firm, histologic features of atypia may also
Weight loss exert an influence on prognosis. High labeling index and
Recurrent vomiting high levels of lactate dehydrogenase are also associated
Dysphagia with poor prognosis.
Bleeding
Anemia Other factors that may influence prognosis are the
presence and number of cytogenetic abnormalities,
54. Which of the following is not a major prognostic hypodiploidy, chromosome 13q and 17p deletion,
factor in multiple myeloma? translocations t(4;14) and t(14;16); circulating plasma
1. Beta 2 microglobulin cells; performance status; as well as serum levels of
2. Labelling Index soluble IL-6 receptor,C-reactive protein, hepatocyte
3. Hypercalcemia growth factor, C-terminal cross-linked telopeptide of
4. Elevated CRP collagen I, transforming growth factor (TGF)-, and
syndecan-1. Microarray profiling and comparative
Ans. 3. Hypercalcemia genomic hybridization have formed the basis for RNA-and
Most other guides have gone wrong with the answer. DXA-based prognostic staging systems, respectively. The
This is a key question, and one of the most vital updates ISS system is the most widely used method of assessing
that has been incorporated into Harrison's "18thEd. prognosis.
Until Harrison's 17th Ed, Salmon-Durie Prognostic System
was in use, which included Hb, serum calcium, Skeletal Durie-Salmon staging system
survey (lytic lesions), Serum M protein, Urinary light chain First published in 1975, the Durie-Salmon staging system
excretion and serum creatinine levels. is still in use:
• stage I: all of
Going with previous editions, the answer so far has been o Hb>10g/dL
cRP, since this was the only, minor (placed under ''Other o normal calcium
Factors") prognostic marker. All other options were o Skeletal survey: normal or single plasmacytoma or
specifically mentioned as important prognostic osteoporosis
determinants. But, Harrison's 18th has dropped this o Serum paraprotein level <5 g/dL if IgG, < 3 g/ dL if
system, and adopted International Staging System(ISS). IgA
Thus, the answer too has changed. With Salmon-Durie o Urinary light chain excretion < 4 g/24h
System being scrapped, Serum Calcium level loses its  stage II: fulfilling the criteria of neither I nor III
status as a major determinant of disease progression: all  stage III: one or more of
other options, except serum Ca++ has been categorically o Hb<8.5g/dL
mentioned as vital prognostic determinants in Harrison's o high calcium > 12 mg/ dL
"18th, thus this is the current answer. o Skeletal survey: Three or more lytic bone lesions
o Serum paraprotein > 7g/dL if IgG, > 5 g/dL if IgA

17
o Urinary light chain excretion > 12g/24h 56. A reticulocyte count >10% will be seen in all of the
 Stages I, II, and III of the Durie-Salmon staging system following except:
can be divided into A or B depending on serum 1. Acute hemorrhage
creatinine: 2. Thallassemia
• A: serum creatinine < 2 mg/dL (< 177 umol/L) 3. Immune Hemolytic anemia
• B: serum creatinine > 2 mg/dL (> 177 umol/L) 4. Rh Incompatibility

55. Dengue Hemorrhagic Fever presents with all Ans. 2. Thallassemia


except: Reduced Reticulocyte Count in Beta Thalassemia
1. Capillary leak Two major features contribute to the pathogenesis of
2. Thrombocytopenia sequelae of β-thalassemia: inadequate β -globin gene
3. Elevated hematocrit production, leading to decreased levels of normal
4. Hypoproteinemia hemoglobin (Hb A), and an imbalance in α- and β -globin
chain production. Selected features of the thalassemias can
Ans. 4. hypoproteinemia be seen in Table 462-6 . In bone marrow, thalassemic
Dengue Hemorrhagic Fever/Dengue Shock Syndrome mutations disrupt the maturation of red blood cells,
Dengue HF is identified by the detection of bleeding resulting in ineffective erythropoiesis; the marrow is
tendencies (tourniquet test, petechiae) or overt bleeding in hyperactive, but the patient has relatively few
the absence of underlying causes such as preexisting reticulocytes and severe anemia. In β -thalassemias, there
gastrointestinal lesions. Dengue shock syndrome, usually is an excess of α-globin chains relative to β - and y-globin
accompanied by hemorrhagic signs, is much more serious chains; α-globin tetramers (α 4) are formed, and these
and results from increased vascular permeability leading inclusions interact with the red cell membrane and
to shock. In mild DHF/DSS, restlessness, lethargy, shorten red cell survival, leading to anemia and increased
thrombocytopenia(<100,000/L), and hemoconcentration ervthroid production. The γ-globin chains are produced in
are detected 2-5 days after the onset of typical dengue increased amounts, leading to an elevated Hb F (α2γ2)- The
fever, usually at the time of defervescence. The 6-globin chains are also produced in increased amounts,
maculopapular rash that often develops in dengue fever leading to an elevated Hb A2(α2δ2) in β -thalassemia.
may also appear in DHF/DSS. In more severe cases, frank
shock is apparent, with low pulse pressure, cyanosis, The infant is born only with Hb F or, in some cases, Hb F
hepatomegaly, pleural effusions, ascites, and in some cases and Hb E (heterozygosity for β°-thalassemia). Eventually,
severe ecchymoses and gastrointestinal bleeding. The patients with p-thalassemia have severe anemia, few
period of shock lasts only 1 or 2 days, and most patients reticulocytes, numerous nucleated red cells, and
respond promptly to close monitoring, oxygen microcytosis, with almost no normal-appearing red cells
administration, and infusion of crystalloid or— in severe on the smear. The hemoglobin level falls progressively to
cases — colloid. The case-fatality rates reported vary <5.0 g/dL unless transfusions are given. The reticulocyte
greatly with case ascertainment and the quality of count is commonly <8% and is inappropriately low
treatment; however, most DHF/DSS patients respond well compared with the degree of anemia because of ineffective
to supportive therapy, and the overall mortality rate at an erythropoiesis. The unconjugated serum bilirubin level is
experienced center in the tropics is probably as low as 1%. usually elevated, but other chemistry values may be
normal at anearly stage. Even if the patient is
The induction of vascular' permeability and shock depends untransfused, eventually there is iron accumulation, with
on multiple factors, including the following: an elevated
1. Presence of enhancing and nonneutralizing serum ferritin level and saturation of transferrin. Bone
antibodies — Transplacental maternal antibody may marrow hyperplasia can be seen on radiographs.
be present in infants <9 months old, or antibody
elicited by previous heterologous dengue infection Laboratory Investigations and Diagnosis of Paroxysmal
may be present in older individuals. T cell reactivity is Nocturnal Hemoglobinuria (PNH)
also intimately involved. The most consistent blood finding is anemia, which may
2. Age—Susceptibility to DHF/DSS drops considerably range from mild to moderate to very severe. The anemia is
after 12 years of age. usually normo-macrocytic, with unremarkable red cell
3. Sex—Females are more often affected than males. morphology; if the MCV is high, it is usually largely
4. Race — Whites are more often affected than blacks. accounted for by reticulocytosis, which may be quite
5. Nutritional status — Malnutrition is protective. marked (up to 20%, or up to 400,000/L). The anemia may
6. Sequence of infection — For example, serotype 1 become microcytic if the patient is allowed to become
followed by serotype 2 seems to be more dangerous iron-deficient as a result of chronic urinary blood loss
than serotype 4 followed by serotype 2. through hemoglobinuria. Unconjugated bilirubin is mildly
7. Infecting serotype — Type 2 is apparently more or moderately elevated, LDH is typically markedly elevated
dangerous than other serotypes. (values in the thousands are common), and haptoglobin is

18
usually undetectable. All these findings make the diagnosis prolonged epistaxis requiring medical attention.
of HA compelling. Menorrhagia is a common manifestation of VWD.
Menstrual bleeding resulting in anemia should warrant an
General examination Jaundice, pallor evaluation for VWD and, if negative ,functional platelet
Other physical finding Spleen may be enlarged; disorders . Frequently ,mild type 1 VWD first manifests
bossing of skull in severe with dental extractions, particularly wisdom tooth
congenital cases extraction ,or tonsillectomy.
Hemoglobin level From normal to severely
reduced Laboratory Diagnosis of von Willebrand Disease
MCV, MCH Usually increased Type aPTT VWF VWFAc FVIIIAc Multimer
Reticulocytes Increased Antige tivity tivity
Bilirubin Increased (mostly un n
conjugated) 1 Nor ↑ ↓ ↓ ↓ Normal distribution,
LDH Increased (up to 10x decreased in
normal with intravascular quantity
hemolysis) 2A Nor ↑ ↓ ↓↓ ↓ Loss of high- and
Haptoglobin Reduced to absent (if intermediate-MW
hemolysis is part multimers
intravascular)_ 2Ba Nor ↑ ↓ ↓↓ ↓ Loss of high-MW
multimers
57. Laboratory finding consistent with von Willebrand 2M Nor ↑ ↓ ↓↓ ↓↓ Normal distribution,
Disease: decreased in
1. Prolonged aPTT quantity
2. Prolonged PT 2N ↑↑ Nor ↓b Nor ↓b ↓ ↓ Normal distribution
3. Bleeding tendency is a must 3 ↑↑ ↓↓ ↓↓ ↓↓ Absent
4. Thrombocytosis
58. Transmural involvement not seen in:
Ans. 1. Prolonged aPTT 1. Ulcerative colitis
Elevated constantly in all forms of vWD. PT is not altered. 2. Crohn's Disease
3. Mesenteric arterial occlusion
von Willebrand Disease 4. Necrotizing enterocolitis
VWD vvVis the most common inherited bleeding disorder.
Estimates from laboratory data suggest a prevalence of Ans. 1. Ulcerative colitis
approximately 1%, but data based on symptomatic UC doesn't involve muscularis propria, and never extends
individuals suggest that it is closer to 0.1% of the beyond mucosa.
population. VWF serves two roles: (1) as the major
adhesion molecule that tethers the platelet to the exposed The distinction between ulcerative colitis and Crohn
sub endothelium; and (2) as the binding protein for FVIII, disease is based, in large part, on the distribution of
resulting in significant prolongation of the FVIII half-life in affected sites and the morphologic expression of disease at
circulation. The platelet-adhesive function of VWF is those sites. Ulcerative colitis is a severe ulcerating
critically dependent on the presence of large VWF inflammatory disease that is limited to the colon and
multimers, while FVIII binding is not. Most of the rectum and extends only into the mucosa and submucosa.
symptoms of VWD are "platelet-like" except in more In contrast, Crohn disease, which has also been referred to
severe VWD when the FVIII is low enough to produce as regional enteritis (because of frequent ileal
symptoms similar to those found in Factor VIII deficiency involvement) may involve any area of the GI tract and is
(hemophilia A). typically transmural.

VWD has been classified into three major types, with four
subtypes of type 2. By far the most common type of VWD is
type 1 disease, with a parallel decrease in VWF protein,
VWF function, and FVIII levels, accounting for at least 80%
of cases. Patients have predominantly mucosal bleeding
symptoms, although postoperative bleeding can also be
seen. Bleeding symptoms are very uncommon in infancy
and usually manifest later in childhood with excessive
bruising and epistaxis. Since these symptoms occur
commonly in childhood, the clinician should particularly
note bruising at sites unlikely to be traumatized and/or

19
Drowsiness Drowsiness
Disorientation Confusion
Dizziness Headache
Headache Coma
Coma
Cardiovascular
↓ Wood pressure ↓ Blood pressure
Ventricular fibrillation Dysrhythmias (related to
(related to hyperkalemia hyperkalemia from
from compensation) compensation))
Warm, flushed skin (related Warm, flushed skin (related
Features That Differ between Crohn Disease and to peripheral vasodilation) to peripheral vasodilation)
Ulcerative Colitis Gastrointestina
Feature Crohn Disease Ulcerative No significant findings Nausea, vomiting, diarrhea,
Colitis Neuromuscular abdominal pain
MACROSCOPIC Seizures
Bowel region Ileum ± colon Colon only Respiratory No significant findings
Distribution Skip lesions Diffuse Hypoventilation with
Stricture Yes Rare hypoxia (lungs are unable Deep, rapid respirations
Wall appearance Thick Thin to compensate when there (compensatory action by
MICROSCOPIC is a respiratory problem) the lungs)
Inflammation Transmural Limited to
Pseudo polyps Moderate mucosa 60. Insulin does not cause:
Ulcers Deep, knife-like Marked 1. Gluconeogenesis in the liver
Lymphoid reaction Marked Superficial, 2. Protein synthesis
Fibrosis Marked broad-based 3. Cellular transport of glucose
Serositis Marked Moderate 4. Hypokalemia
Granulomas Yes (-35%) Mild to none
Fistulae/ sinuses Yes Mild to none Ans. 1. Gluconeo gene sis in the liver
No Principal Actions of Insulin.
No
CLINICAL Rapid (seconds)
Perianalfistula Yes (in colonic No Increased transport of glucose, amino acids, and K+ into
Fat/ vitamin disease; No insulin-sensitive cells
malabsorption Yes Yes Intermediate (minutes)
Malignant potential With colonic No Stimulation of protein synthesis
Recurrence after involvement Yes Inhibition of protein degradation
surgery Common Activation of glycolytic enzymes and glycogen synthase
Toxic megacolon No Inhibition of phosphorvlase and gluconeogenic enzymes
Delayed (hours)
59. Increased irritability of the skeletal muscles is Increase in mRN'As for lipogenic and other enzymes
classically seen in The actions of insulin on adipose tissue; skeletal, cardiac,
1. Metabolic acidosis and smooth muscle; and the liver are summarized:
2. Metabolic alkalosis
3. Respiratory acidosis Effects of Insulin on Various Tissues
4. Respiratory alkalosis Adipose tissue
Increased glucose entry
Ans. 2. Metabolic alkalosis Increased fatty acid synthesis
Metabolic alkalosis Increased glycerol phosphate synthesis
• pH greater than 7.45 Increased triglyceride deposition
• PC02 normal or may be inc. b/c of compensating Activation of lipoprotein lipase
• HC03 greater than 26 Inhibition of hormone-sensitive lipase
• urine PH greater than 6 Increased K+ uptake
Muscle
Lewis's Medical Surgical Nursing By Chintamani Increased glucose entry
writes: Increased glycogen synthesis
Respiratory (↑ PaCO2) Metabolic (↓HCO3- ) Increased amino acid uptake
Neurologic Increased protein synthesis in ribosomes

20
Decreased protein catabolism (3) Cerebellar hemorrhage (occipital headache, vomiting,
Decreased release of gluconeogenic amino acids gaze paresis, and inability to stand);
Increased ketone uptake (4) Basilar artery thrombosis (neurologic prodrome or
Increased K+ uptake warning spells, diplopia, dysarthria, vomiting, eye
Liver movement and corneal response abnormalities, and
Decreased keto genesis asymmetric limb paresis); and
Increased protein synthesis (5) Subarachnoid hemorrhage (precipitous coma after
Increased lipid synthesis headache and vomiting).
Decreased glucose output due to decreased
gluconeogenesis, increased glycogen synthesis, and 62. A 14 year old boy fell off a bicycle and injured his
increased glycolysis head. However, he had a good Glasgow Coma
General Scalelevel when he was admitted for observation. 3
Increased cell growth hours after admission, he became non-responsive to
Ref: review of Medical physiology 23Ed Ganong pain and unconscious. What is the possible diagnosis?
1. Extradural hemorrhage
61. Which is the commonest area of intracranial 2. Subdural hemorrhage
hypertensive bleed? 3. Subarachnoid hemorrhage
1. Basal ganglia 4. Intracerebral hemorrhage
2. Brain stem
3. Subdural hemorrhage Ans. 1. Extradural hemorrhage > B. Subdural hemorrhage
4. Cerebellum EDH is safer option, though data provided here seems
insufficient to differentiate thes two etiologies. A CT Scan
Ans. 1. Basal ganglia is mandatory.
Hypertensive Intraparenchymal Hemorrhage A "lucid interval" of several minutes to hours before coma
Pathophysiology supervenes is most characteristic of epidural hemorrhage
Hypertensive intraparenchymal hemorrhage (i.e. Extradural hemorrhage), but have been described
(hypertensive hemorrhage or hypertensive intracerebral frequently for Subdural hemorrhage with up to one-third
hemorrhage) usually results from spontaneous rupture of of patients have a lucid interval.See below:
a small penetrating artery deep in the brain. The most
common sites are the basal ganglia (especially the Acute subdural hematomas typically arise from venous
putawen), thalamus, cerebellum, and pons. When sources, often the bridging veins locatedimmediately
hemorrhages occur in other brain areas or in under the dura mater. As the brain volume decreases with
nonhypertensive patients, greater consideration should be age, traction on these venousstructures increases and even
given to hemorrhagic disorders, neoplasms, vascular minor head trauma in the elderly can lead to a subdural
malformations, and other causes. The small arteries in hematoma.
these areas seem most prone to hypertension-induced Approximately 33% of patients with an acute subdural
vascular injury. bleed will experience a lucid interval after theevent, which
The hemorrhage may be small or a large clot may form and is followed bit obtundation. Subdural bleeding is typically
compress adjacent tissue, causing herniation and death. slower than epidural bleeding due to their different
Blood may dissect into the ventricular space, which sources .Small subdural bleeds are asymptomatic and
substantially increases morbidity and may cause often do not require evacuation.
hydrocephalus.
Most hypertensive intraparenchymal hemorrhages Diffuse Axonal Injury
develop over 30-90 min, whereas those associated with A special type of deep white matter lesion consists of
anticoagulant therapy may evolve for as long as 24-48 h. widespread mechanical disruption, or shearing, of axons at
Within 48 h macrophages begin to phagocytize the the time of impact. Most characteristic are small areas of
hemorrhage at its outer surface. After 1-6 months, the tissue injury in the corpus callosum and dorsolateral pons.
hemorrhage is generally resolved to a slitlike orange cavity The presence of widespread axonal damage in both
lined with glial scar and hemosiderin-laden macrophages, hemispheres, a state called diffuse axonal injury (DAI), has
cause the greatest difficulty in coma diagnosis. been proposed to explain persistent coma and the
The most common categories are: vegetative state after closed head injury, but small
(1) Basal ganglia and thalamic hemorrhage (acute but not ischemic-hemorrhagic lesions in the midbrain and
instantaneous onset, vomiting, headache, hemiplegia, thalamus are as often the cause. Only severe shearing
and characteristic eye signs); lesions that contain blood are visualized by CT, usually in
(2) Pontine hemorrhage (sudden onset, pinpoint pupils, the corpus callosum and centrum semiovale; however,
loss of reflex eye movements and corneal responses, selective imaging sequences of the MRI can demonstrate
ocular bobbing, posturing, hyperventilation, and such lesions throughout the white matter.
excessive sweating);responses, ocular nobbing,
posturing, Hyperventilation, and excessive sweating); Diagnosis
21
CT without contrast initially shows a low-density mass shifts; this circumstance in an older patient is suggested by
over the convexity of the hemisphere (Fig. 378-5). a "hypernormal" CT scan with fullness of the cortical sulci
Between 2 and 6 weeks after the initial bleeding the and small ventricles. Infusion of contrast material
hemorrhage becomes isodense compared to adjacent brain demonstrates enhancement of the vascular fibrous capsule
and may be inapparent. Many subdural hematomas that surrounding the collection. MRI reliably identifies
are several weeks in age contain areas of blood and subacute and chronic hematomas.
intermixed serous fluid. Bilateral chronic hematomas may
fail to be detected because of the absence of lateral tissue
63. Atonic bladder is seen in
1. Spinal shock
2. Spinal injury above the level of sacral segments
3. Brainstem injury
4. Transection of the afferent nerves from the bladder

Ans. 1. Spinal shock


Sacral Cord/Conus Medullaris
The conus medullaris is the tapered caudal termination of
the spinal cord, comprising the lower sacral and
singlecoccygeal segments. The distinctive conus syndrome
consists of bilateral saddle anesthesia (S3-S5), prominent
bladder and bowel dysfunction (urinary retention and
incontinence with lax anal tone), and impotence. The Human infection begins when a female anopheline
bulbocavernosus (S2-S4) and anal (S4-S5) reflexes are
mosquito inoculates plasmodial sporozoites from its
absent (Chap. 367). Muscle strength is largely preserved. salivary gland during a blood meal. These microscopic
By contrast, lesions of the cauda equina, the nerve roots motile forms of the malarial parasite are carried rapidly
derived from the lower cord, are characterized by low back
via the bloodstream to the liver, where they invade hepatic
and radicular pain, asymmetric leg weakness and sensory parenchymal cells and begin a period of asexual
loss, variable areflexia in the lower extremities, and reproduction. By this amplification process (known as
relative sparing of bowel and bladder function. intrahepatic or preerythrocytic schizogony or merogony),
a single sporozoite eventually may produce from 10,000 to
Immediate after effects of acute spinal cord injury >30,000 daughter merozoites. The swollen infected liver
During the immediate after effects of acute spinal cord cell eventually bursts, discharging motile merozoites into
injury, there is a state of spinal shock; during spinal shock the bloodstream. These merozoites then invade the red
the patient has a flaccid paralysis below the level of the blood cells (RBCs) and multiply six- to twentyfold every
spinal lesion; no reflexes are obtained, sensation is not 48-72 h. When the parasites reach densities of ~50/~ L of
perceived and the bladder loses its capacity to contract
blood (~100 million parasites in the blood of an adult), the
and is in a state of retention overflowing when fully symptomatic stage of the infection begins. In P. vivax and
stretched; this is an atonic bladder and may last for days, P, ovale infections, a proportion of theintrahepatic forms
weeks or months
do not divide immediately but remain dormant for a
1. this occurs because the inhibiting effects of the higher period ranging from 3 weeks to a year or longer before
central nervous system has been cut off from the reproduction begins. These dormant forms, or hypnozoites
lower cord , are the cause of the relapses that characterize infection
2. as spinal shock passes off, reflexes return (flexor with these two species.
reflexes of skeletal muscle return first in 3 days to 3
weeks following injury) and become hyperactive; Characteristics of Plasmodium Species Infecting
spasticity sets in and the bladder begins to show
Humans
evidence of contractions even when the bladder isn't
full

64. Infective stage of malaria to human?


1. Sporozoite
2. Trophozoite
3. Schizont
4. Gametocytes

Ans. 1. Sporozoite

22
Finding for Indicated Species
Characteristic P. P. vivax P. P.
falcipar ovale malariae
um
Duration 5.5 8 9 15
ofintrahepatic
phase(days)
Number 30,000 10,000 15,000 15,000
ofmerozoitesrele
ased
perinfectedhepat
ocyte The C wave may reflect the carotid pulsation in the neck
Duration 48 48 50 72 and/or an early systolic increase in right atrial pressure as
oferythrocytic the right ventricle pushes the closed tricuspid valve into
cycle(hours) the right atrium (bulging/ coving up).
Red cell Younger Reticulocy Reticul Older cells
preference cells tes and ocytes The venous waveform is divided into several distinct
(but cells peaks. The a wave reflects right atrial presystolic
can up to 2 contraction and occurs just after the electrocardiographic
invade weeks old P wave, preceding the first heart sound (S1).
cells
of all  A prominent a wave is seen in patients with reduced
ages) right ventricular compliance; a cannon a wave occurs
Morphology Usually Irregularl Infecte Band with atrioventricular (AV) dissociation and right atrial
only y derythr orrectang contraction against a closed tricuspid valve. In a
ring shapedlar ocytes, ular patient with a wide complex tachycardia, the
Formsb; ge rings enlarge forms
banana- andtropho d and oftrophoz appreciation of camion a waves in the jugular venous
Shapedg zoites; ovalwit oites waveform identifies the rhythm as ventricular in
ametocy enlarged h tufted common origin. The a wave is not present with atrial
tes erythrocyt ends; fibrillation.
es; Schuffn  The x descent defines the fall in right atrial pressure
Schuffner’ er’s after inscription of the a wave.
s dots dots  The c wave interrupts this x descent and is followed by
Pigment color Black Yellow- Dark Brown- a further descent.
brown brown black  The v wave represents atrial filling (atrial diastole)
Ability to cause No Yes Yes No and occurs during ventricular systole. The height of
relapses the v wave is determined by right atrial compliance as
well as the volume of blood returning to the right
65. C wave in JVP is caused by: atrium either antegrade from the cavae or retrograde
1. Atrial contraction through an incompetent tricuspid valve. In patients
2. Buldging of the Tricuspid Valve with TR, the v wave is accentuated and the subsequent
3. Filling of the Blood against closed valve fall in pressure (y descent) is rapid. With progressive
4. Idioventricular rhythm degrees of TR, the v wave merges with the c wave, and
the right atrial and jugular vein waveforms become
Ans. 2. Buldging of the Tricuspid Valve "ventricularized."
 The y descent, which follows the peak of the v wave,
can become prolonged or blunted with obstruction to
right ventricular inflow, as may occur with tricuspid
stenosis (TS) or pericardial temponed.

66. Tetany is seen in all of the following except:


1. Hypocalcemia
2. Hypomagnesemia
3. Hypoparathyroidism
4. Metabolic acidosis

Ans. 4. metabolic acidosis

23
All other options execp metabolic acidosis has certain • Infiltration by metastases or systemic diseases
association with hypocalcemia/ tatany. Alkalosis of nil • Autoimmune
types, hyperaldosteronism, hypocalcemia, and Reduced parathyroid function
hypokalemia may present with tetani. • Hypomagnesaemia
• Activating CaSR mutations
Clinical and Laboratory Findings of Metabolic High Parathyroid Hormone Levels (Secondary
Alkalosis Hyperparathyroidism)
With metabolic alkalosis, changes in CXS and peripheral Vitamin D deficiency or impaired 1,25(OH)2D
nervous system function are similar to those of production/action
hypocalcemia; symptoms include mental confusion; • Nutritional vitamin D deficiency (poor intake or
obtundation; and a predisposition to seizures, paresthesia, absorption)
muscular cramping, tetany, aggravation of arrhythmias, • Renal insufficiency with impaired 1,25(OH)2D
and hypoxemia in chronic obstructive pulmonary disease. production
Related electrolyte abnormalities include hypokalemia and • Vitamin D resistance, including receptor defects
hypophosphatemia. Parathyroid hormone resistance syndromes
• PTH receptor mutations
Clinical and Laboratory Findings of Respiratory • Pseudohypoparathyroidism (G protein mutations)
Alkalosis Drugs
• (From, Harrison's Principles Of internal Medicine 18th • Calcium chelators
Edition) • Inhibitors of bone resorption (bisphosphonates,
The hyperventilation syndrome may be disabling. plicamycin)
Paresthesia; circumoral numbness; chest wall • Altered vitamin D metabolism (phenytoin,
tightness or pain; dizziness; inability to take an ketoconazole)
adequate breath; and, rarely, tetany may be Miscellaneous causes
sufficiently stressful to perpetuate the disorder. • Acute pancreatitis
• Acute rhabdomyolysis
• (From, Davidson's Principles and Practice of Medicine • Hungry bone syndrome after parathyroidectomy
21st Ed) • Osteoblastic metastases with marked stimulation of
Clinical features are those associated with the cause, bone formation (prostate cancer)
but there is frequently also agitation associated with Abbreviations: CaSR, calcium sensor receptor; PTH,
perioral and digital tingling, due to a reduction in parathyroid hormone.
ionised calcium concentration caused by increased
binding of calcium to albumin in the alkalotic ECF. In Causes of Hypercalcemia
severe cases, Trousseau's sign and Chvostek's sign Excessive PTH production
maybepositive ,and tetany or seizures may • Primary hyperparathyroidism (adenoma, hyperplasia,
develop(p.766). rarely carcinoma)
• Tertiary hyperparathyroidism (long-term stimulation
Causes of Tetany of PTH secretion in renal insufficiency)
• The usual cause of tetany is lack of calcium. An excess • Ectopic PTH secretion (very rare)
of phosphate (high phosphate-to-calcium ratio) can • Inactivating mutations in the CaSR (FHH)
also trigger the spasms. Milk-and-alkali tetany is an • Alterations in CaSR function (lithium therapy)
example of this imbalance. Hypercalcemia of malignancy
• Under function of the parathyroid gland can lead to • Overproduction of PTHrP (many solid tumors)
tetany. • Lytic skeletal metastases (breast, myeloma)
• Low levels of carbon dioxide cause tetany by altering Excessive l,25(OH)2D production
the albumin binding of calcium such that the • Granulomatous diseases (sarcoidosis, tuberculosis,
ionised (physiologically influencing) fraction of silicosis)
calcium is reduced; the most common reason for • Lymphomas
low carbon dioxide levels is hyperventilation. • Vitamin D intoxication
• Low levels of magnesium can lead to tetany. Primary increase in bone resorption
• Hyperthyroidism
Low Parathyroid Hormone Levels • Immobilization
(Hyperparathyroidism) Excessive calcium intake
Parathyroid agenesis • Milk-alkali syndrome
• Isolated • Total parenteral nutrition
• DiGeorge syndrome Other causes
Parathyroid destruction • Endocrine disorders (adrenal insufficiency,
• Surgical pheochromocvtoma, VIPoma)
• Radiation
24
Medications (thiazides, vitamin A, antiestrogens) In untreated patients or in patients in whom therapy has
not adequately controlled virus replication, after a variable
67. In view of ART therapy all are true except: period, usually measured in years, the CD4+ T cell count
1. ART started when CD4 count <200/µL in adults falls below a critical level (<200/L) and the patient
2. ART started when CD4+ T cell count <500/ µL in becomes highly susceptible to opportunistic disease. For
adults this reason, the CDC case definition of AIDS includes all
3. ART initiation is independent of CD4 count in infants HIV-infected individuals with CD4+ T cell counts below
4. ART is indicated in pregnant women with HIV this level (Table 189-1). Patients may experience
constitutional signs and symptoms or may develop an
Ans. 1. ART started when CD4 count <200/µL in adults opportunistic disease abruptly without any prior
Indications for the Initiation of Antiretroviral Therapy in symptoms, although the latter scenario is unusual. The
Patients with HIV Infection depletion of CD4+ T cells continues to be progressive and
I. Acute infection syndrome unrelenting in this phase. It is not uncommon for CD4+ T
II. Chronic infection cell counts inthe untreated patient to drop as low as 10/L
A. Symptomatic disease (including HIV- or even to zero. In countries where cART and prophylaxis
associated nephropathy) and treatment for opportunistic infections are readily
B. Asymptomatic disease accessible to such patients, survival is increased
1. CD4+ T cell count <500/ La dramatically even in those patients with advanced HIV
2. Pregnancy disease. In contrast, untreated patients who progress to
III. Post exposure prophylaxis this severest form of immunodeficiency usually succumb
'This is an area of controversy. Some experts would treat to opportunistic infections or neoplasm’s (see below).
everyone regardless of CD4+ T cell count.
Source :Guidelines for the Use of Antiretroviral Agents in Abbreviation; PGL, progressive generalized
HIV-infected Adults and Adolescents, USPHS. lymphadenopathy
Clinical Categories of HIV Infection
Clinical Categories Category A: Consists of one or more of the conditions
listed below in an adolescent or adult (>13 years) with
CD4+ T A B C AIDS- documented HIV infection. Conditions listed in categories
Cell Asymptomatic, Symptomatic, Indicator B and C must not have occurred.
Categories Acute Not A or C Conditions Asymptomatic HIV infection
(Primary) HIV Conditions Persistent generalized lymphadenopathy
or PGL Acute (primary) HIV infection with accompanying illness
>500/ L Al Bl CI or history of acute HIV infection
200-499/ A2 B2 C2 Category B: Consists of symptomatic conditions in an HIV-
L infected adolescent or adult that are not included among
<200/L A3 B3 C3 conditions listed in clinical category C and that meet at
least one of the following criteria:
In Pediatric HIV, (1) The conditions are attributed to HIV infection or are
Initiation of Therapy indicative of a defect in cell-mediated immunity ;or
HIV-infected children with symptoms (clinical category A, (2) the conditions are considered by physicians to have a
B, or C) or with evidence of immune dysfunction (immune clinical course or to require management that is
category 2 or 3) should be treated with anti-retroviral complicated by HIV infection. Examples include, but are
therapy, regardless of age or viral load. not limited to, the following:
Children <1 yr of age are at high risk for disease Bacillary angiomatosis
progression, and immunologic and virologic tests to Candidiasis, oropharyngeal (thrush)
identify those likely to develop rapidly progressive disease Candidiasis, vulvovaginal; persistent, frequent, or poorly
are less predictive than in older children. Therefore, such responsive to therapy
infants should be treated with anti-retroviral agents as Cervical dysplasia (moderate or severe)/cervical
soon as the diagnosis of HIV carcinoma in situ
infection has been confirmed, regardless of clinical or Constitutional symptoms, such as fever (38.5°C) or
immunologic status, or viral load. diarrhea lasting >"1 month
Hairy leukoplakia, oral
Data surest that HIV-infected infants who are heated Herpes zoster (shingles), involving at least two distinct
before the age of 3 months control their HIV infection episodes or more than one dermatome
better than infants whose anti-retroviral therapy started Idiopathic thrombocytopenic purpura
later than 3 mo of age. Listeriosis
Pelvic inflammatory disease, particularly if complicated by
Advanced HIV Disease tuboovarian abscess

25
Peripheral neuropathy 68. Prophylaxis of Pneumocystis carnii pneumonia
Category C: Conditions listed in the AIDS surveillance case includes:
definition. 1. Clotrimoxazole
Candidiasis of bronchi, trachea, or lungs 2. Penicillin
Candidiasis, esophageal 3. Vancomycin
Cervical cancer, invasive 4. Fluoroquinolones
Coccidioidomycosis, disseminated or extrapulmonary
Cryptococcosis, extrapulmonary Ans. 1. Clotrimoxazole
Cryptosporidiosis, chronic intestinal (>1 month's Pathogenesis and Pathology Pneumocystis Infection;
duration) Studies over the past several years have shown that
Cytomegalovirus disease (other than liver, spleen, or Pneumocystis commonly colonizes patients who are
nodes) immune suppressed or who have chronic obstructive
Cytomegalovirus retinitis (with loss of vision) pulmonary disease. This colonization elicits an
Encephalopathy, HIV-related inflammatory response and is associated with a decline in
Herpes simplex: chronic ulcer(s) (>1 month's duration); or lung function.
bronchitis, pneumonia, or esophagitis
Histoplasmosis, disseminated or extra pulmonary The host factors that predispose to the development of PcP
Cryptosporidiosis, chronic intestinal (>1 month's include defects in cellular and humoral immunity. The risk
duration) of PcP among HIV-infected patients rises markedly when
Cytomegalovirus disease (other than liver, spleen, or circulating CD4+ T cell counts fall below 200/ L. Other
nodes) persons at risk for PcP are patients receiving
Cytomegalovirus retinitis (with loss of vision) immunosuppressive agents (particularly glucocorticoids)
Encephalopathy, HIV-related for cancer and organ transplantation; those receiving
Herpes simplex: chronic ulcer(s) (>1 month's duration); or biologic agents such as infliximab and etanercept for
bronchitis, pneumonia, or esophagitis rheumatoid arthritis and inflammatory bowel disease;
Histoplasmosis, disseminated or extrapulmonary children with primary immunodeficiency diseases; and
Isosporiasis, chronic intestinal (>1 month's duration) premature malnourished infants.
Kaposi's sarcoma
Lymphoma, Burkitt's (or equivalent term)
Lymphoma, primary, of brain
Mycobacterium avium complex or M. kansasii,
disseminated or extrapulmonary
Mycobacterium tuberculosis, any site (pulmonary or
extrapulmonary)
Mycobacterium, other species or unidentified species,
disseminated or extrapulmonary
A B
Pneumocystis jiroveci pneumonia
Pneumonia, recurrent Prevention for Pneumocystis Infections
Progressive multifocal leukoencephalopathy Prophylaxis is indicated for HIV-infected patients with
Salmonella septicemia, recurrent CD4+ T cell counts of <200/(µL or a history of
Toxoplasmosis of brain oropharyngeal candidiasis and for both HIV-infected and
Wasting syndrome due to HIV non-HIV-infected patients who have recovered from PcP.
Prophylaxis may be discontinued in HIV-infected patients
Mechanisms of CD4+ TCell Dysfunction and Depletion once CD4+ T cell counts have risen to >200/ µL and
Direct Mechanisms Indirect Mechanisms remained at that level for 3 months. Primary prophylaxis
Loss of plasma Aberrant intracellular' signaling guidelines for immune compromised hosts not infected
membrane integrity due events with HIV are less clear.
to Autoimmunity TMP-SMX is the drug of choice for primary and secondary
viral budding Innocent bystander killing of prophylaxis (Table 207-2). This agent also provides
Accumulation of viral antigen-coated protection against toxoplasmosis and some bacterial
unintegrated viral DNA cells infections. Alternative regimens are available for
Interference with cellular Apoptosis individuals intolerant of TMP-SMX (Table 207-2). Although
RXA processing Inhibition of lymphopoiesis there are no specific recommendations for preventing the
Intracellular gp120-CD4 Activation-induced cell death spread of Pneumocystis in health care facilities, it seems
autofusion events Elimination of HIV-infected cells prudent to prevent direct contact between patients with
Syncytia formation by virus-specific PcP and other susceptible hosts.
immune responses

26
Prophylaxis of Pneumiocytosis
Drug(s), Dose, Route Comments
•First Choice
TMP-SMX, 1 DS tablet or 1 SS tablet qd PO TMP-SMX can be safely reintroduced for treatment of some
patients who have had mild to
moderate side effects.
•Other Agents
Dapsone, 50 mg bid or 100 mg qd PO -
Dapsone, 50 mg qd PO; plus pyrimethamine, 50mg weekly Leucovorin prevents bone marrow toxicity from
PO; plus leucovorin, 25 mg weeklyPO pyrimethamine.

Dapsone, 200 mg weekly PO; plus Leucovorin prevents bone marrow toxicity from plus
pyrimethamine, 75 mg weekly PO; plus pyrimethamine.
leucovorin, 25 mg weekly PO
Pentamidine, 300 mg monthly via Respirgard IInebulizer Adverse reactions include cough and bronchospasm.
Atovaquone, 1500 mg qd PO -
TMP-SMX, 1 DS tablet three times weekly PO TMP-SMX can be safely reintroduced for treatment of some
patients who have had mild tomoderate side effects.

Treatment of Pneumocystosis
Drug(s), Dose, Route Adverse Effects
• First Choice
TMP-SMX (TMP: 5 mg/kg; SMX: 25 mg/kg)q6-8h PO or IV Fever, rash, cytopenias, hepatitis, hyperkalemia, GI
disturbances
• Other Agents
TMP, 5 mg/kg q6-8h; plus dapsone, 100 mg qd PO Hemolysis (G6PD deficiency), methemoglobinemia, fever, rash,
GI disturbances
Atovaquone, 750 mg bid PO Rash, fever, GI and hepatic disturbances
Clindamycin, 300-450 mg q6h PO or 600 mg q6-8h IV; plus Hemolysis (G6PD deficiency), methemoglobinemia,
primaquine, 15-30 mg qd PO rash,colitis, neutropenia
Pentamidine, 3-4 mg/kg qd IV Hypotension, azotemia, cardiac arrhythmias, pancreatitis,
dysglvcemias, hypocalcemia, neutropenia, hepatitis
Trimetrexate, 45 mg/m2 qd IV; plus leucovorin/ 20 mg/kg Cytopenias, peripheral neuropathy, hepatic disturbances
q6h PO or IV
• Adjunctive Agent
Prednisone, 40 mg bid x 5 d, 40 mg qd x 5 d, 20mgqdxlld; PO Immunosuppression, peptic ulcer, hyperglycemia, mood
or lV changes, hypertension

69. A 3 day old infant was brought in with complaints of being restless and feed intolerant. He has a heart rate of 250
beats per minute, and ECG shows a QRS complex of duration 0.07 sees, with no discernible p waves. Which of the
following is a likely diagnosis?
1. SVT
2. PSVT with aberrant conducting channels
3. Ventricular Tachycardia
4. Atrial fibrillation

Ans. 2. PSVT with aberrant conducting channels


Rate beyond 200 bpm, narrow QRS, and "absent" (undermined) P waves, all suggest Reentrant SIT.
AF does show P (f) waves: VT will have wide QRS complexes. SVT (in general) will have definite P waves.

ECG Findings in AVNodal Reentrant Tachycardia

27
The APC initiating AVXRT is characteristically followed by a long PR interval consistent with conduction via the slow pathway.
AVXRT is manifest typically as a narrow QRS complex tachycardia at rates that range from 120 to 250 beats/min. The QRS-P
wave pattern associated with typical AVXRT is quite characteristic, with simultaneous activation of the atria and ventricles
from the reentrant AV nodal circuit.
The P wave frequently is buried inside the QRS complex mid either will not be visible or will distort the initial or terminal
portion of the QRS complex. Because atrial activation originates in the region of the AV node, a negative deflection will be
generated by retrograde atrial depolarization when recording ECG leads II, III, or AVF.

Ventricular Tachycardia
ECG showing AV dissociation (arrows mark P waves), wide QRS >200 ms, superior frontal plane axis, slurring of the initial
portion of the QRS, and large S wave in V-all clues to the diagnosis of ventricular tachycardia.

ECG Clues Supporting the Diagnosis of Ventricular Tachycardia

AV dissociation (atrial capture, fusion beats)


QRS duration >140 ms for RBBB type V1> morphology; V1>160 ms for LBBB type V1 morphology
Frontal plane axis -90° to I800
Delayed activation during initial phase of the QRS complex
LBBB pattern-R wave in V1, V2>40 ms
RBBB pattern— onset of R wave to nadir of S >100 ms
Bizarre QRS pattern that does not mimic typical RBBB or LBBB QRS complex
Concordance of QRS complex in all precordial leads
RS or dominant S in V6 for RBBB VT
Q wave in V6 with LBBB QRS pattern
Monophasic R or biphasic qR or R/ S in V1 with RBBB pattern

Atrial Fibrillation

Ventricular rate vary between 120 and 160 beats per minute, in some patients it can be >200 beats per
minute.
Counterclockwise right AFLrepresents -80% of all AFL withsuperiorly directed activation of theinteratrial septum, which
producesthe saw-toothed appearance of the P waves in ECG leads II, III, and aVF. Clockwise rotation of the same right atrial
circuit producespredominantly positive P waves in leads II, III, and aVF (Fig. 233-4). Macro reentrant left AFL also
maydevelop, albeit much less commonly.
28
The mechanism for AF initiation and maintenance, although still debated, appears to be a complex interaction between drivers
responsible for the initiation and the complex anatomic atrial substrate that promotes the maintenance of multiple wavelets of
(Micro)re-entry. The drivers appear to originate predominantly from the atrialized musculature that enters the pulmonary
veins and represent either focal abnormal automaticity or triggered firing that is somewhat modulated by autonomic
influences. Sustained forms of micro reentry as drivers also have been documented around the orifice of pulmonary veins;

70. Limiting factor to ORS stool output is


1. 5 ml/kg/hr
2. 10 ml/kg/hr
3. 15 ml/kg/hr
4. 20 ml/kg/hr

Ans. 1. 5ml/kg/hour, probably


Truly speaking, I didn't quite get, what exactly has been asked?
Is it asking about the rate of stool output, beyond which ORS is risky (i.e. IV fluid becomes mandatory) or it simply asked the
maximum allowable dose of ORS in diarrhoea?
You can go through Dr. OP Ghai 7th Ed page-265
Or, check IAP Book, which says 5ml/kg/hr is the highest rate at which ORS can be safe, beyond which, superadded IV fluid is
necessary. It writes:

Deficit fluid therapy for Severe Dehydration'(Plan C)


Infants (<1 year) Older child (>1 year)
Volume of Ringer's lactate 30 ml/kg body wt. within first I hour, 30 ml/kg body weight within hour,
followed by 70 ml/kg body weight over followed by 70 ml/kg body ' weight
next 5 hours over next 2 ½ hours
Monitoring Assess for improvement every 1-2 hours:
• If not improving, give IV infusion more rapidly
• Encourage oral feeding by giving ORS 5 ml/kg/hour, along with IV fluids, as soon
as the child is able to drink
Reassess hydration status:
• After 6 hours (infants) and 3 hours (Older children) assess hydration status and
choose appropriate plan

71. A child can make of tower of three cubes at:


1. 12 months
2. 15 months
3. 18 months
4. 24 months

Ans. 2. 15 months
Builds tower of 2-3 cubes (Uses objects in combination) by 15 months
Builds tower of 6 cubes (Requires visual, gross, and fine motor coordination) by 22 months
• 15 MO Motor: Walks alone; crawls up stairs Adaptive: Makes tower of 3 Cubes; makes a line with crayon; inserts raisin in
bottle
• 18 MO Motor: Runs stiffly; sits on small chair; walks up stairs with one hand held; explores drawers and wastebaskets
Adaptive: Makes tower of 4 Cubes; imitates scribbling; imitates vertical stroke; dumps raisin from bottle
• 24 MO Motor: Runs well, walks up and down stairs, one step at a time; opens doors; climbs on furniture; jumps Adaptive:
Makes tower of 7 Cubes (6 at 21 mo); scribbles in circular pattern; imitates horizontal stroke; folds paper once imitatively
• 30 MO Motor: Goes up stairs alternating feet Adaptive: Makes tower of 9 Cubes; makes vertical and horizontal strokes, but
generally will not join them to make cross; imitates circular stroke, forming closed figure
• 36 MO Motor: Rides tricycle; stands momentarily on one foot Adaptive: Makes tower of 10 Cubes; imitates construction of
"bridge'' of 3 cubes; copies circle; imitates cross.

Developmental Milestones in the First 2 Yr of Life

MILESTONE AVERAGE AGE DEVELOPMENTAL IMPLICATIONS


OFATTAINMENT(MO)

29
GROSS MOTOR
Holds head steady while sitting 2 Allows more visual interaction
Pulls to sit, with no head lag 3 Muscle tone
Brings hands together in midline 3 Self-discovery of hands
Asymmetric tonic neck reflex gone 4 Can inspect hands in midline
Sits without support 6 Increasing exploration
Rolls back to stomach 6.5 Truncal flexion, risk of falls
Walks alone 12 Exploration, control of proximity to parents
Runs 16 Supervision more difficult
FINE MOTOR
Grasps rattle 3.5 Object use
Reaches for objects 4 Visuomotor coordination
Palmar grasp gone 4 Voluntary release
Transfers object hand to hand 5.5 Comparison of objects
Thumb-finger grasp 8 Able to explore small objects
Turns pages of book 12 Increasing autonomy during book time
Scribbles 13 Visuomotor coordination
Builds tower of 2 cubes 15 Uses objects in combination
Builds tower of 6 cubes 22 Requires visual, gross, and fine motor coordination
COMMUNICATION AXD LANGUAGE
Smiles in response to face, voice 1.5 More active social participant
Monosyllabic babble 6 Experimentation with sound, tactile sense
Inhibits to "no" 7 Response to tone (nonverbal)
Follows one-step command withgesture 7 Nonverbal communication
Follows one-step commandwithout gesture 10 Verbal receptive language(e.g./'Give it to me")
Says “mama” or “dada” 10 Expressive language
Points to objects 10 Interactive communication
Speaks first real word 12 Beginning of labeling
Speaks 4-6 words 15 Acquisition of object and personal names
Speaks 10-15 words 18 Acquisition of object and personal names
Speaks 2-word sentences(e.g./'Mommy 19 Beginning grammaticization, corresponds with50+
shoe") word vocabulary
COGNITIVE
Stares momentarily at spot whereobject 2 Lack of object permanence (out of sight, out ofmind)
disappeared [e.g., yam ball dropped]
Stares at own hand 4 Self-discovery, cause and effect
Bangs 2 cubes 8 Active comparison of objects
Uncovers toy (after seeing it hidden) 8 Object permanence
Egocentric symbolic play (e.g./pretends to 12 Beginning symbolic thought
drink from cup)
Uses stick to reach toy 17 Able to link actions to solve problems
Pretend play with doll (e.g., gives 17 Symbolic thought
doll bottle)

72. Which is the least common complication of acute post streptococcalglomerulonephritis in child?
1. Heart failure
2. Encephalopathy
3. Hyperkalemia
4. Bleeding diathesis

Ans. 4. Bleeding diathesis


Complications of Acute Rheumatic Endocarditis
Severe mitral insufficiency may result in cardiac failure that may be precipitated by progression of the rheumatic process, the
onset of atrial fibrillation, or infective endocarditis. The effects of chronic mitral insufficiency may become manifest after many
years and include right ventricular failure and atrial and ventricular' arrhythmias.

30
Renal failure, consequent hyperkalemia and • Amino acid and organic acid disorders (eg, maple
encephalopathy are also widely recognized complications. syrup urine disease, propionic acidemia,
methylmalonic acidemia, tyrosinosis, glutaric aciduria,
Scute Rheumatic Carditis (Pancarditis) is now history. 3-hydroxy-3-methylglutaric aciduria)
There is conspicuous evidence that, it's primarily an  Systemic disease (eg, sepsis, burns, cardiogenic shock,
endocarditis only. Resultant myocardial failure (so-called respiratory distress syndrome)
"Myocarditis") resolves instantly with valve prosthesis).
74. Nuchal translucency is seen in
73. Neonatal hypoglycemia is associated with 1. Open neural tube defects
1. Adrenal Insufficiency 2. Trisomy 21
2. Glucocorticoid exces 3. Trisomy 13
3. Pancreatic aplasia 4. Trisomy 15
4. Postmaturity
Ans. 2. Trisomy 21
Ans. 1. Adrenal Insufficiency Increased Nuchal translucency/ fold thickness is most
Both Adrenal (Medulla) insufficiency and Congenital commonly associated with Down's syndrome; but, is also
Adrenal Hyperplasias (Cortex) may lead to Neonatal found in Turner syndrome, Trisomy IS, Trisomy 13 and
hypoglycemia. Triploidy. So, Trisomy 21 has to be the pick.
Other options do not decrease serum glucose level. So,
itsthe answer of exclusion. Nuchal translucency
SYMPTOMS OF HYPOGLYCEMIA The translucent area measured (the nuchal translucency)
• Jitteriness is due to a blockage of fluid in the developing fetal
• Tremors lymphatic system. Progressive increase in the width of the
• Apnea translucent area during the 11 to 14 week measurement
• Cyanosis period is thus indicative of congenital lymphedema
• Limpness/ lethargy
• Seizures Fetuses at risk of Down's syndrome tend to have a higher
INFANTS AT RISK FOR HYPOGLYCEMIA amount of fluid around the neck.
Diminished production
 Limited glycogen This is only useful to measure between 11 and 14 weeks of
• SGA gestation, when the fetal lymphatic system is developing
• Prematurity and the peripheral resistance of the placenta is high.
• Birth stress
• Glycogen storage disorders After 14 weeks the lymphatic system is likely to have
 Limitedgluconeogenesis developed sufficiently to drain away any excess fluidand
• SGA changes to the placental circulation will result in a drop in
• Inborn errors peripheral resistance. So after this time any abnormalities
Increased utilization causing fluid accumulation may seem to correct
 Hyperinsulinism themselves and can thus go undetected by nuchal
• IDM scanning.
• Beckwith-Wiedemann syndrome
• Nesidioblastosis or pancreatic adenoma Other chromosomal defects that cause a thicker nuchal
• Erythroblastosis fetalis translucency are
• Exchange transfusion, chlorpropamide, • Turner syndrome
benzothiazides, • Trisomy 18
• I2-sympathomimetics, malpositioned UA catheter • Trisomy 13
Unknown Mechanism • Triploidy
 LGA infants who are not IDM
 Sepsis
 Polycythemia or hyperviscosity syndrome
 Congenital hypopituitarism
The causes of hypoglycemia in older infants, children, and
teenagers include:
• Poisonings/drugs (eg, ethanol, isoniazid, insulin,
propranolol, salicylates, oral hypoglycemics,
pentamidine, quinine, disopyramide, unripe ackee
fruit, Vacor).
• Liver disease (eg, Reye syndrome, hepatitis, cirrhosis,
hepatoma)
31
Abdominal and particularly vaginal sonography are useful
diagnostic techniques for evaluating endometrium.
Endometrial thickness can be measured and the echo
pattern of the different parts of endometrium analysed.
During normal cycles a triple line sign is typical for the late
proliferative and periovulatory phase. In the secretory
phase the endometrium is echogenic and posterior
enhancement of echoes can be seen. It is also useful to
control the effects of hormonal treatments or the
endometrium using sonography. In postmenopausal
women endometrial thickness of 4-5 mm or more is
abnormal andfurther evaluations are indicated. In
endometrial cancer sonography is a reliable way to
estimate myometrial invasion.

The signs of secretory endometrium included absent


Nuchal fold thickness
triple-line sign, hyper echoic functional layer, and strong
A nuchal scan is a sonographic prenatal screening scan
posterior acoustic enhancement. When all of these US
(ultrasound) to help identity higher risks of chromosomal
signs were clearly depicted in combination, the accuracy of
defects including Down's syndrome in a fetus, particularly
diagnosis was 93% (30 patients). The accuracy of
for older women who have higher risks of such
diagnosis for all patients in the study was 76% (73
pregnancies. High thickness measurements are also
patients). The relative distributions of each US sign in
associated with congenital heart defect
proliferative and secretory endometrium were compared.
Despite complicating extrinsic factors such as uterine
75. Triple line sign is seen in the endometrium in
leiomyomas and intrinsic factors such as the transition
which of the following phase?
period between endometrial phases, US staging of the
1. Secretory phase
endometrium is a useful diagnostic technique that allows
2. Proliferative phase
serial examinations of the endometrium.
3. Endometriosis
4. Endometrial carcinoma
76. First changes occurs during treatment with iron
deficiency anemia
Ans. 2. Proliferative phase
1. Reticulocyte count increase
Ultrasound evaluation of the Endometrium
2. Hemoglobin increase
• After menstruation the endometrium is demonstrated
3. Ferritin level
as a single echogenic line.
4. RBC indices
• The estrogen biosynthesis by the dominant follicle
leads to stimulation of the endometrium with growth
Ans. 1
of the glands and stroma (i.e. Proliferative Phase). This
Causes of iron deficiency
change is visualized as the "triple-line". The interface
Increased demand for iron and/or hematopoiesis
between the two leaves of the endometrium is the
• Rapid growth in infancy or adolescence
central echogenic line. The two outer lines represent
• Pregnancy
the basalis of the developing endometrium.
• Erythropoietin therapy
• The endometrium appears hypoechoic compared to
Increased iron loss
the echogenic lines. The thickness of the endometrium
• Chronic blood loss
can reach 6 to 10 mm, around the time of ovulation,
• Menses
which represents a two to three fold increase in its
• Acute blood loss
original thickness.
• Blood donation
• Phlebotomy as treatment for polycythemia vera
Decreased iron intake or absorption
• Inadequate diet
• Malabsorption from disease (sprue, Crohn's disease)
• Malabsorption from surgery (postgastrectomy)
• Acute or chronic inflammation

Generic name Tablet (iron Elixir (iron content),


An ultrasound image of a thin endometrium portraying a content), mg mg in 5 mL
single echogenic line and Visualization of the triple-line
Ferrous sulfate 325 (65) 300 (60)
sign of the endometrium at the period ovulation
195 (39) 90(18)
32
Extended release 525 (105)  Thymoma (MC)
325 (107)  Teratoma
 Lymphoma
Ferrous fumarate 195 (64) 100 (33)
 Thyroid lesions
Ferrous 325 (39) 300 (35)
 Parathyroid tumors.
gluconate 150 (150) 100(100)
Posterior mediastinum
Polysaccharide 50(50) 100(100)
 Neurogenic tumors (schwannoma, neurofibroma)
iron
(MC)
 Lymphoma
77. A 30 Year Old Patient comes to casualty with  Gastroenteric hernia.
nausea,hematemesis, diarrhea. Chest X-Ray finding is Middle mediastinum
given below. On Evaluation patient is found to have  Bronchogenic cyst
metabolic acidosis. What is your diagnosis  Pericardial cyst
 Lymphoma.

79. Adenoma sebaceum in aldolescent is seen in


1. Neurofibromatosis
2. Tuberous sclerosis
3. von Hippel-Lindau syndrome
4. All of them

Ans. 2
Tuberous Sclerosis (Bourneville's Disease)
 Tuberous sclerosis is characterized by triad of
cutaneous lesions seizures and mental retardation.
 The cutaneous lesions include adenoma sebaceum
(facial angiofibromas), ash leafshaped hypopigmented
macules (best seen under ultraviolet illumination with
1. Aspirin Toxicity
a Wood’s lamp), shagreen patches (yellowish
2. Iron Toxicity
thickenings of the skin over the lumbosacral region of
3. Lead Toxicity
theback) and depigmented nevi.
4. Alcoholic Toxicity
 Tuberous sclerosis patients are at increased risk of
developing ependymoma childhood astrocytomas, of
Ans. 2. Iron poisoning, abdominal x-ray
which >90% are subependymal giant cell
Iron poisoning presents with GI symptoms (nausea,
astrocytomas. These are benign neoplasms that may
vomiting, hematemesis, diarrhea) due to the iron induced
develop in the retina or along the border of the lateral
damage to the GI mucosa. Toxic iron levels also cause
ventricles. They may obstruct the foramen of Monro
hypoperfusion (veno/capillary dilation) and mitochondrial
and produce hydrocephaly Rhabdomyomas of the
damage, both causing a metabolic acidosis. Since iron
myocardium and angiomyomas of the kidney, liver,
tablets are radio-opaque they can be seen in the stomach
adrenals and pancreas may also occur.
on AXR. Diagnosis with serum iron levels and treat with an
 Treatment is symptomatic. Anticonvulsants for
iron chelator such as deferoxamine.
seizures, shunting for hydrocephalus, and behavioral
and educational strategies for mental retardation are
78. Tumor commonly seen in the anterior
the mainstays of management. Severely affected
mediastinum
individuals generally die before age 30.
1. Neurofibroma
 Mutations in either the TSC-1 gene at 9q or the TSC-2
2. Metasatic carcinoma
gene at 16p are associated with tuberous sclerosis.
3. Teratoma
These genes encode tuberins, proteins that modulate
4. Schwannoma
the GTPase activity of other cellular signaling proteins.
Ans. 3
80. Treatment of choice in cryptococcal meningitis
Common Mediastinal Tumors
1. Fluconazole
Superior mediastinum
2. Nystatine
 Lymphoma
3. Amphotericin B
 Thymoma
4. Clotrimazole
 Thyroid lesions
 Metastatic carcinoma
Ans. 3
 Parathyroid tumors.
Amphotericin B and other polyenes, such as nystatin, bind
Anterior mediastinum
to ergosterol in fungal cell and increase membrane
33
permeability. The imidazoles and triazoles, such as palsy region
itraconazole and fluconazole, inhibit 14-α-sterol
demethylase, prevent ergosterol synthesis and lead to the 83. AIHA and pernicious anemia are example of which
accumulation of 14-α-methylsterols. The allylamines, such type hypersensitivity?
as naftifine and terbinafine, inhibit squalene epoxidase 1. Type 1
andp;revent ergosterol synthesis. The echinocandins (e.g. 2. Type 2
capofungin) inhibit the formation of glucans in the fungal 3. Type 3
cell wall. 4. Type 4

81. Which is not seen with scarcoidosis? Ans. 2. Type 2


1. Hypercalcemia
2. Osteomalacia Ref—Ananthanarayan and Paniker 9th ed, p. 166; Robbins
3. Diabetes insipidus 9th ed, p. 202-203
4. Addison’s disease AIHA and pernicious anemia are example of type 2
hypersensitivity reaction.
Ans. 2 It was asked from our explanation part of question no 62
 Sarcoidosis is an inflammatory disease characterized (2014 paper) of Sure Success in
by the presence of noncaseating granulomas. The WBPGMEE book so go through the chart carefully.
disease is often multisystem and requires the presence
of involvement in two or more organs for a specific 84. False positive D-xylose test is seen in all except
diagnosis. The finding of granulomas is not specific for 1. Ascites
sarcoidosis, and other conditions known to cause 2. Renal failure
granulomas must be ruled out. These conditions 3. Hepatic failure
include mycobacterial and fungal infections, 4. Blind loop syndrome
malignancy and environmental agents such as
beryllium. Ans. 3
 Respiratory complaints including cough and dyspnea False positive D-xylose test seen in the following condition
are the most common presenting symptoms. 1. Third space collection (ascites/ pleural effution)
 Symptoms related to cutaneous and ocular disease are 2. Delayed gastric emptying
the next two most common complaints. Skin lesions 3. Rapid intestinal transit
are often nonspecific. 4. Renal insufficiency
 Nonspecific constitutional symptoms include fatigue, 5. Bacterial overgrowth
fever, night sweats and weight loss. Fatigue is perhaps 6. Vomiting
the most common constitutional symptom that affects 7. Blind loop syndrome
these patients.
Schilling test
82. Which of these is pathognomonic for motor neuron Part 1 test result Part 2 test result Diagnosis
disease? Normal - Normal or
1. Extensor plantar response vitamin B12
2. Sensory loss deficiency
3. Fasciculation Low Normal Problems with IF
4. Bowel and bladder involvement production, e.g.
pernicious
Ans. 3 anemia
Type UMN LMN Low Low Malabsorption
degeneration degeneration (terminal ileum)
Amyotrophic Yes Yes
lateral sclerosis 85. Revised criteria for rheumatoid arthritis include
(ALS) the following except
Primary lateral Yes No 1. Morning stiffness
sclerosis (PLS) 2. Asymmetrical arthritis
Progressive No Yes 3. Rheumatoid nodule
muscular atrophy 4. Arthritis of three or more joints
(PMA)
Progressive No Yes, bulbar Ans. 3
bulbar palsy region Classification criteria for rheumatoid arthritis
(PBP) Score
Pseudobulbar Yes, bulbar No Joint involvement 1 large joint 0
34
(shoulder, elbow, (%) (range),
hip, knee, ankle) years
2-10 large joints 1 Esophagus 57 0.5 67
1-3 small joints 2 Stomach 213 29 30.1 (10-61)
(MCP, PIP, thumb Small 520 13 41.7 (21-84)
IP, MTP, wrists) intestine
4-10 small joints 3 Colon 84 39 45.8 (27-71)
>10 joints (at 5 Pancreas 132 36 40.8 (16-60)
least 1 small Lung 17 15
joint) Testis 4.5 9 8.6 (3-20)
Serology Negative RF and 0 Breast 15.2 54 37(9-48)
negative ACPA Uterus 16 9
Low-positive RF 2 Ovary 27 21 28(4-57)
or low-positive Cervix 1.5 10 34.3 (23-54)
anti-CCP
antibodies (3
times ULN) Size of polyp and malignancy
High-positive RF 3 Size of polyp Malignant potential
or high-positive < 1.5 cm Negligible
anti0CCP 1.5-2.5 cm Intermediate
antibodies (>3 >2.5 cm Substantial
times ULN)
Acute-phase Normal CRP and 0 88. BPH is most commonly involved which lobe of the
reactants normal ESR prostate?
Abnormal CRP or 1 1. Anterior
abnormal ESR 2. Posterior
Duration of <6 weeks 0 3. Median
symptoms 4. Lateral
≥6 weeks 1
Ans. 3
86. Scirrhous carcinoma of stomach is best diagnosed Lobes
by? The "lobe" classification is more often used in anatomy.
1. CT Anterior lobe (or isthmus) Roughly corresponds to
2. Ba meal part of transitional zone
3. Endoscopy Posterior lobe Roughly corresponds to
4. Clinical examination peripheral zone
Lateral lobes Spans all zones
Ans. 3 Median lobe (or middle Roughly corresponds to
Symptom’s of gastric carcinoma lobe) part of central zone
S-Slient Endoscopy with multiple
O-Obstruction biopsy and brush cytology
L-Lump is the investigation of • BPH most commonly arise from median lobe
I-Incidious onset choice for all type of gastric • Carcinoma prostate most commonly affects posterior
D-Dysphagia carcinoma lobe.
87. A girl presented with pigmentation of lip with The common drugs used in BPH and their side effects
intestinal polyp. Her younger sister suffers from same Drug Dosage Mechanism Side effect
disease. Most common diagnosis:
Finast 5 mg 5a-reductase Impotence,
1. Villous adenoma
eride once inhibitor decreased libido,
2. Peutz-Jeghers syndrome
daily decreased semen
3. Carcinoid
quantity at
4. Melanoma
ejaculation,
decreased semen
Ans. 2
prostate specific
Risk ratio compared to the general population,
antigen,
lifetime frequency, mean age and age range of
gynecomastia
malignancies related to Peutz-Jeghers syndrome
(rare)
Organ Risk ratio Frequency Mean age
35
Dutast 0.5 mg 5a-reductase Impotence,
eride once inhibitor decreased libido, 90. Thyroid surgery not indicated in:
daily decreased semen 1. Thyroid carcinoma
quantity at 2. For cosmesis
ejaculation, 3. Pain
decreased semen 4. Toxic goiter
prostate specific
antigen, Ans. 3
gynecomastia AMES classification of thyroid cancers
(rare) Low mortality High mortality
Teraz 1 mg α1,-adrenergic Asthenia, risk risk
osin once receptor hypotension, A:Age Men; <41 years of Men: >41 years of
daily to antagonist dizziness, age age
start; somnolence Women: <51 Women: >51
may years of age years of age
increase M: Metastases Absence of Presence of
up to 10 distant distant
mg/day metastases metastases
Doxaz 1 mg 1-adrenergic Orthostatic E: Extent of Intrathyroidal Extrathyroidal
osin once receptor hypotension, primary tumor papillary cancer papillary cancer
daily to antagonist fatigue, dyspnea (confined to Follicular cancer
start; thyroid) with major
may Follicular cancer capsular
increase with minor involvement
up to 8 capsular
mg once involvement
daily S: Size of tumor Primary tumor Primary tumor 5
Tams 0.4/0.8 α1-adrenergic Dizziness, rhinitis, <5 cm in cm in diameter
ulosin mg once receptor abnormal diameter (regardless of
daily antagonist ejaculation extent of disease)
Alfuzo 2.5 mg α1-adrenergic Fatigue, edema,
sin tid/5 receptor rhinitis, headache, 91. A 23 year old female was worked up for infertility
mg antagonist upper respiratory Below is the USG finding. What is ur diagnosis
bid/10 tract infection
mg once
daily
Saw 160 mg Mixed Aggravate chronic
palme twice gastrointestinal
tto daily disease such as
peptic ulcer

89. In blunt trauma abdomen organ most commonly


suffered:
1. Spleen
2. Liver
3. Cecum
4. Pancreas

Ans. 1
Blunt Trauma Abdomen
Solid organs usually involved 1. Müllerian agenesis
1. Spleen (most common) 2. Uterus didelphys
2. Liver 3. Bicornuate uterus
3. Kidney 4. Septated uterus
In addition to these organs, blunt force to the upper
abdomen may fracture the pancreas. Answer 3. Bicornuate Uterus
Which is susceptible to injury because of its position
overlying the vertebral column. Class I: Müllerian agenesis (absent uterus).

36
Uterus is not present, vagina only rudimentary or absent.
The condition is also called Mayer-Rokitansky-Kuster- 93. Projectile vomiting is seen in
Hauser syndrome. The patient with MRKH syndrome will 1. Hypertrophic pyloric stenosis
have primary amenorrhea. 2. Esophageal atresia
Class II: Unicornuate uterus (a one-sided uterus). 3. Intussusception
Only one side of the Müllerian duct forms. The uterus has a 4. Cholera
typical "penis shape" on imaging systems.
Class III: Uterus didelphys, also uterus didelphis (double Ans. 1
uterus).  Cholera is characterized by an acute onset of copious
Both Müllerian ducts develop but fail to fuse, thus the watery diarrhea and vomiting without abdominal
patient has a "double uterus". This may be a condition with cramps or fever.
a double cervix and a vaginal partition (v.i.), or the lower  The stools are colorless with small flecks of mucus
Müllerian system fused into its unpaired condition. See ("rice-water") and are sometimes described as having
Triplet-birth with Uterus didelphys for a case of a woman a fishy odor.
having spontaneous birth in both wombs with twins.  At first, children may be restless or extremely thirsty,
Class IV: Bicornuate uterus (uterus with two horns). but if fluid and electrolyte losses are not replaced, they
Only the upper part of that part of the Müllerian system may become lethargic or unconscious.
that forms the uterus fails to fuse, thus the caudal part of  Other signs of dehydration may rapidly manifest,
the uterus is normal, the cranial part is bifurcated. The including poor skin turgor, sunken eyes, dry mouth
uterus is "heart-shaped". and tongue, no urine output, delayed capillary refill,
Class V: Septated uterus (uterine septum or partition). rapid or weak pulse and low blood pressure.
The two Müllerian ducts have fused, but the partition  Severe dehydration, metabolic acidosis, and
between them is still present, splitting the system into two hypokalemia can occur in 4-12 h.
parts. With a complete septum the vagina, cervix and the The fluid losses may be so rapid that the child quickly
uterus can be partitioned. Usually the septum affects only develops hypovolemic shock, hypoglycemia, coma and
the cranial part of the uterus. A uterine septum is the most seizures and is at risk of dying within a few hours of
common uterine malformation and a cause for onset
miscarriages. It is diagnosed by medical image techniques, Treatment—ORS (with rice)
i.e. ultrasound or an MRI. MRI is considered the preferred
modality due to its multiplanar capabilities as well as its 94. Comprehensive neck dissection refers to
ability to evaluate the uterine contour, junctional zone, and 1. Radical neck dissection
other pelvic anatomy. A hysterosalpingogram is not 2. Extended neck dissection
considered as useful due to the inability of the technique to 3. Posterior triangle dissection
evaluate the exterior contour of the uterus and distinguish 4. Selective neck dissection
between a bicornuate and septate uterus.
Ans. 1
92. Sistrunk’s operation is done in Neck dissection operations are classified according to the
1. Thyroglossal cyst cervical lymphatic regions that are resected.
2. Dermoid cyst Selective neck dissection (SND) is done for NO necks (no
3. Thyroid nodule clinical evidence of neck nodes) or for very limited cervical
4. Branchial cleft cyst metastases. Central neck dissection encompasses only
level VI.
Ans. 1 Comprehensive or therapeutic neck dissection involves
Anatomical clues to the diagnosis of neck masses surgical clearance of levels I-V and may either be a radical
Location/morphology of Possible cause (RND) or modified (MND) neck dissection. RND includes
mass resection of sternocleidomastoid muscle (SCM) and
Midline Thyroglossal duct cyst, accessory nerve (XIn) and internal jugular vein (IJV). MND
dermoid, teratoma preserves SCM and/or XIn and/or IJV. MND type I entails
Anterior to Branchial cleft remnant preservation of 1/3, usually XIn; MND type II entails
sternocleidomastoid muscle anomalies preservation of 2/3, usually XIn and IJV; with MND type III
Posterior to Lymphangioma/cystic all 3 structures are preserved. MND type II is most
sternocleidomastoid muscle hygroma commonly done, and is oncologically acceptable in the
Near thyroid Diffuse thyroid enlargement absence of adherence of cervical nodal metastases to XIn
or thyroid nodule or IJV.
Preauricular Parotid or submandibular Extended neck dissection includes additional lymphatic
gland pathology groups (parotid, occipital, level VI, mediastinal,
Lymph node clusters Inflammatory or neoplastic retropharyngeal) or nonlymphatic structures (skin,
lymphadenopathy

37
muscle, nerve, blood vessels, etc.) not usually included in a 2. Brochial asthma
comprehensive neck dissection. 3. Wheal and flare reaction aftering of antigen
- The neck is conventionally divided into 6 levels; level VII 4. Aspergillus culture +ve from sputum
is in the superior mediastinum.
- Level 1 is bound by the body of the mandible above, the Ans. 4
stylohyoid muscle posteriorly, and the anterior belly of the ABPA is a hypersensitivity reaction to Aspergillus species,
contralateral digastric muscle anteriorly. The revised commonly found fungi in soil. It occurs almost exclusively
classification uses the posterior margin of the in patients with asthma or cystic fibrosis. The disease is
submandibular gland as the boundary between levels I and the result of immunologic response to the endoluminal
II as it is clearly identified on ultrasound CT or MRI. Level I growth of species.
is subdivided into level Ia, (submental triangle) which is The diagnosis of ABPA is based on clinical, laboratory and
bound by the anterior bellies of the digastric muscles and radiologic criteria. Primary criteria include asthma,
the hyoid bone, and level Ib (submandibular triangle). radiologic evidence of pulmonary infiltration, positive skin
test to A. fumigatus, eosinophilic precipitating Ab to A.
95. Smoking causes which of the following fumigatus, elevated IgE, elevated A.fumigatus- specific IgE
transformation and IgG and central bronchiectasis. A diagnosis of ABPA is
1. Transitional metaplasia nearly certain when six of these eight criteria are fulfilled.
2. Squamous metaplasia But, many of these findings are nonspecific. Only central
3. Neuroendocrine metaplasia bronchiectasis is nearly pathognomomic and essential to
4. Mucinous metaplasia diagnosis to most cases.
The symptoms include cough, hemoptysis, fever, pleuritic
Ans. 2 pain, wheezing and dyspnea.
Original tissue Stimulus Metaplasstic
tissue 98. A 60 year old male is brought to the Emergency
Ciliated columnar Cigaret smoke Squamous ward with cyanosis, mottling, altered mental status,
epithelium of epithelium tachycardia, and hypotension. Chest X-ray is given
bronchial tree below what could be the diagnosis
Transition Trauma of Squamous
epithelium of calculus epithelium
bladder
Fibrocolagenous Chronic trauma Bone (osseous)
tissue tissue
Esophageal Gastric acid Columnar
squamous epithelium
epithelium
Columnar Vitamin A Squamous
glandular deficiency epithelium
epithelium

96. 40-year-old male present with ascites varies


varicose vain with caput medusa in abdomen along
1. Lead Poisoning
with pedal edema. Occlusion most probably occurs in
2. Ruptured Pericardium
1. IVC
3. Rupture of Aortic Aneurysm
2. Portal vain
4. Duodenal Bleed
3. Hepatic artery
4. Interstitial vain
Ans. 3. Acute aneurysm:
Ruptured Aortic Aneurysm On A Chest X Ray
Ans. 2
There are 3 categories of portal hypertension:
99. Which fistula is most likely to close with
 Prehepatic (thrombosis in splenic or portal vein). conservative management?
 Intrahepatic (postsinusoidal-veno-occlusive disease, 1. Gastric
sinusoidal-cirrhosis, presinusoidal-schistosomiasis) 2. Duodenal
 Posthepatic (heart failure, constrictive pericarditis cor 3. Esophageal
pulmonale also known as RV enlargement from 4. Ileal
pulmonary disease)
Ans. 3
97. Not a major criteria fro ABPA  Reported incidence of esophageal anastomotic leak is
1. High IgE 13-16%.
38
 Most leaks are clinically insignificant and can be
managed with adequate drainage and nutritional Ans. 1. Beta oxidation of fatty acids
support. Normal myocardium utilises more of fatty acids than
 If retropleural approach is undertaken for primary glucose for energy production. Oxidation of fatty acids
esophageal anastomosis with a patent mediastinal requires more of oxygen per ATP molecule generated. If
drain is in place, upto 95% of anastomotic leaks close beta oxidation of fatty acids is inhibited, the metabolism
spontaneously. shifts to glucose utilisation which uses less oxygen per
 Even when transthoracic repair is followed by molecule of ATP generated, hence it is useful to the
disruption and pleural space contamination, adequate ischemic myocardium.
drainage can usually allow spontaneous closure of
leak. 103. Prolotherapy is used for the treatment of
1. Osteoarthritis
100. In TIPS, shunt done between: 2. Thrombocytopenia
1. Hepatic artery and splenic vein 3. Rickets
2. Hepatic artery and portal vein 4. Leucopenia
3. Bile duct and portal vein
4. Hepatic vein and portal vein Ans. 1
Prolotherapy involves the injection of an irritant solution
Ans. 4 into the area where connective tissue has been weakened
 What does TIPS mean? or damaged through injury or strain. Many solutions are
 TIPS stands for transjugular intrahepatic used, including dextrose, lidocaine (a commonly used local
portosystemic shunt: anesthetic), phenol, glycerine, or cod liver oil extract. The
o Transjugular—through the jugular vein in injection is given into joint capsules or
the neck. where tendon connects to bone. Many points may require
o Intrahepatic—within the liver. injection. The Injected solution causes the body to heal
o Portosystemic—from the portal vein to itself through the process of inflammation and repair. In
the hepatic vein (The portal vein carries the case of weakened or torn connective tissue, induced
blood into the liver. The hepatic vein inflammation and release of growth factor at the site of
drains blood from the liver). injury may result in a 30-40% strengthening of the
o Shunt—a metal tube covered with fabric attachment points, although strong scientific evidence
that allows flow from one blood system supporting this is lacking. It is now used in osteoarthritis
to another. and tennis elbow.
 Indications
 Variceal bleeding (preventive in recurrent cases) 104. A 4-month-old child had silvery gray hair, light-
 Variceal bleeding (for treatment in acute setting) colored skin, recurrent chest infections,
hepatosplenomegaly. Light microscopy of hair showed
 Refractory ascites and hepatic hydrothorax
characteristic large aggregates of pigment granules
 Portal hypertensive gastropathy
distributed irregularly along the hair shaft. Peripheral
 Hepatorenal syndrome
blood smear examination did not show giant granules
 Lower gastrointestinal and stomal varices
in granulocytes. Enlarged hyperpigmented basal
melanocytes with sparsely pigmented adjacent
keratinocytes were seen on the skin biopsy specimen.
101. Comby's sign is seen in?
What is this syndrome
1. Measles
1. Gricelli Syndrome
2. Lichen planus
2. Apert’s Syndrome
3. Moniliasis
3. Cat eye Syndrome
4. Pemphigus
4. Wegener’s Granulomatosis
Ans. 1. The Comby sign is a clinical sign of early measles in
Ans 1 Griscelli syndrome - is a rare autosomal recessive
which thin, whitish patches are seen on
disorder characterized by albinism (hypopigmentation)
the gums and buccal mucosa due
with immunodeficiency. Cat Eye Syndrome is a rare
to desquamation of epithelial cell
condition caused by The short arm (p) and a small section
of the long arm (q) of humanChromosome 22 being
102. Trimetazidine is an antianginal agent which acts
trisomic) or tetrasomic . The term "Cat Eye" syndrome was
by inhibition of:
coined due to the particular appearance of the
1. Beta oxidation of fatty acids
vertical colobomas in the eyes of some patients.
2. Anaerobic metabolism of glucose
3. Calicum channels
105. Nicorandil is a:
4. Beta adrenergic receptors
1. Calcium channel blocker
39
2. Potassium channel blocker Sitosterolemia is confirmed by demonstrating an increase
3. Sodium channel blocker in the plasma level of sitosterol using gas chromatography.
4. Potassium channel activator The hypercholesterolemia does not respond to HMG-CoA
reductase inhibitors; however, bile acid sequestrates and
Ans. 4. Potassium channel activator cholesterol-absorption inhibitors, such as ezetimibe, are
Nicorandil is an anti anginal agent without any significant effective in reducing plasma sterol levels in these patients.
hemodynamic effect.
108. A 25-year-old man presents to the ER with a 3-
106. A 36 year old moderate drinker presents with month history of intermittent pounding headaches,
diabetic ketoacidosis and acute pancreatitis. What is sweating, and palpitations. He denies any symptoms of
the likely findings on examination. Serum Triglyceride depression or anxiety. On examination, he is a thin
level 1800 mg/dl gentleman, BP 240/120, heart rate 110/minute,
1. Tendon xanthomas thyroid not enlarged. There is no prior history of
2. Lipemia retinalis hypertension.The most likely diagnosis is which of the
3. Eruptive tuberous xanthomas following?
4. Milky Serum 1. Carcinoid syndrome
2. Pheochromocytoma
Ans. 3 Eruptive xanthomas are associated with 3. Aldosteronoma
hypertriglyceridemia, particularly that associated with 4. Renal artery stenosis
types I, IV, and V (high concentrations of VLDL and
chylomicrons). They appear in secondaryhyperlipidemias, Ans. 2The classic triad of pheochromocytoma is sweating,
particularly in diabetes. headache, and palpitations. When these are associated
Acute Pancreatitis : triglyceride levels higher than 500 with hypertension, they have a sensitivity and specificity of
mg/dL >90% for the diagnosis. Paroxysms are not a component of
Eruptive Xanthomas triglyceride count above 1000 mg/dL aldosterone secreting tumors or renal artery stenosis.
Lipaemia Retinalis above 2000 mg /dl Hyperthyroidism and panic attacks would be in the
Milky Serum above 2500 mg/dl differential diagnosis, but thyroid cancer is not associated
with hypertension.
107. A 30 year old patient presents with elevated
plasma levels of LDL cholesterol. Patients develop 109. A 74-year-old female with a history of
tendon xanthomas as well as premature hypertension and hypothyroidism is admitted with
atherosclerosis and Episodes of hemolysis are present. easy bruising, guaiac positive stools, and anemia (Hgb
The hypercholesterolemia does not respond to HMG- 8.1 g/dL). Screening coagulation tests reveal a
CoA reductase inhibitors; however, aezetimibe, is prolonged activated partial thromboplastin time
effective in reducing plasma sterol levels in these (aPTT) with a normal prothrombin time (PT) and
patients. What is the diagnosis platelet count. What is the next step in the diagnosis of
1. Familial Hypercholesterolemia this woman’s problem?
2. Polygenic Hypercholesterolemia 1. Perform upper and lower endoscopy with biopsies.
3. Sitosterolemia 2. Check factors II, VII, IX, and X levels.
4. ApoA-V Deficiency 3. Check factors XI, VII, IX, and VIII levels
4. Check an aPTT 1:1 mix with normal plasma and 1-
Ans. 3. hour incubation
Most Probable in Question
Sitosterolemia is inherited as a rare autosomal recessive Ans. 4.A 1:1 mixing study is done when the PT or PTT is
condition. It has been shown to result from mutations in prolonged. The patient’s plasma is mixed with normal
either of two adjacent and oppositely oriented genes plasma and the abnormal test is repeated. If the mixing of
(ABCG5 and ABCG8) located in chromosome 2 normal plasma corrects the abnormal test (PT or PTT),
It is characterized by hyperabsorption and decreased then a factor deficiency is suggested; otherwise, an
biliary excretion of dietary sterols leading inhibitor is suspected. Similarly, an incubated mixing study
to hypercholesterolemia, tendon and tuberous xanthomas, is done 1 hour (and occasionally 2 hours)
premature development of atherosclerosis, and abnormal after mixing of the patient plasma with normal plasma. It is
hematologic and liver function test results. used to differentiate a lupus anticoagulant from factor
Patients with sitosterolemia usually have elevated plasma inhibitors.
levels of LDL cholesterol. Patients develop tendon
xanthomas as well as premature atherosclerosis and can 110. A 61-year-old female is diagnosed with
be mistaken for FH patients. Episodes of hemolysis, steoporosis by a screening dual-energy x-ray
presumably secondary to the incorporation of plant sterols absorptiometry (DEXA) scan. Which of these is a risk
into the red blood cell membrane, are a distinctive clinical factor for postmenopausal osteoporosis?
feature of this disease. 1. Lack of exercise
40
2. Obesity Acanthosis nigricans is a raised, velvety, tan skin lesion
3. Multiparity commonly seen on the back of the neck, in the axilla, and
4. Late menopause the intertriginous areas. Acanthosis nigricans isassociated
with hyperinsulinemia and is a sign that the patient is at
Ans. 1Risk factors for postmenopausal osteoporosis significant risk for prediabetes and frank diabetes. Up to
include cigarette smoking, thinness, early menopause 30% of patients with suspected PCOS have prediabetes,
(natural or surgical), nulliparity, northern European and 8% are frank type II diabetics.
heritage, some medications (e.g., glucocorticoids), high
alcohol or caffeine intake, low dietary calcium 113. A19-year-old college student is found to have an
consumption, and a family history of osteoporosis. Dietary elevated serum calcium on routine physical
calcium and vitamin D supplementation and exercise examination. She has a family history of hypercalcemia
(preferably weight bearing) are the classic lifestyle that has not resulted in any known symptoms. Further
changes recommended to prevent or treat osteoporosis. workup reveals a slightly elevated serum parathyroid
hormone with depressed levels of serum phosphate. A
111. A 5-year-old girl presents for evaluation of breast 24- hour urine calcium excretion is obtained and is
development, history of multiple bone fractures, and low. Which of the following is the correct diagnosis?
vaginal bleeding. Physical examination is notable for 1. Familial hypocalciuric hypercalcemia (FHH)
“cafe au lait” spots on her skin, tanner stage 2 breasts, 2. Primary hyperparathyroidism
and she appears tall for her age. What is the most 3. Secondary hyperparathyroidism
likely cause of precocious puberty in this child? 4. Tertiary hyperparathyroidism
1. Acromegaly
2. McCune-Albright syndrome Ans. 1. FHH, or familial benign hypercalcemia, is a rare
3. Ovarian cyst condition characterized by asymptomatic or mildly
4. Ingestion of her mother’s oral contraceptives symptomatic hypercalcemia. It is inherited as anautosomal
dominant trait and the parathyroid glands are usually
Ans. 2. The McCune-Albright syndrome is due to a G- normal in size. The basis for the development of FHH
protein mutation in the alpha-subunit that causes appears to be mutations in the calcium-sensing receptor
constitutive stimulatory activity of the tissues. Affected gene which regulates the parathyroid gland setpoint and
tissues are autonomously active. McCune-Albright modulates the extracellular calcium concentration. The
syndrome classically consists of a triad of cafe au lait skin condition may be mistaken for primary
spots, cystic bone lesions (polyostotic fibrous dysplasia), hyperparathyroidism because, in both conditions, the
and autonomous endocrine hyperfunction resultingin serum calcium and parathyroidhormone levels are
sexual precocity. Adominant ovarian cyst develops elevated with a concomitant low serum phosphate. The
independent of stimulation by gonadotropins and it distinction is made by obtaining a 24-hour urine calcium
secretes estradiol, resulting in sexual precocity and excretion level. In patients with FHH, the urine calcium
skeletal maturity. level is low, whereas in primary hyperparathyroidism the
Therefore, the finding of elevated estradiol levels and level is high. The distinction is important, as patients with
suppressed or undetectable gonadotropins is diagnostic. primary hyperparathyroidism benefit from surgery and
Affected individualsalso may have hyperthyroidism, those with FHH do not.
hypercortisolism, pituitary gigantism, or acromegaly.
114. A42-year-old male with extensive Crohn’s disease
112. A39-year-old obese female presents with undergoes a near complete resection of the ileum.
irregular menstrual periods, mild acne and hirsutism, Adeficiency of which of the following vitamin is likely
and acanthosis nigricans on the nuchal fold, axilla, and to result?
intertriginous areas (inner upper thighs). You suspect 1. Niacin
PCOS. Which of the following laboratory tests would be 2. Thiamine
most important to perform to rule out a likely 3. Vitamin B12
confounding diagnosis? 4. Vitamin C
1. 2-hour oral glucose tolerance test
2. Testosterone Ans. 3. The distal small bowel (ileum) is the site of
3. Prolactin absorption of fat-soluble vitamins (vitamins A, D, E, and K)
4. Dehydroepiandrosterone sulfate as well as vitamin B12. Vitamin B12 binds with intrinsic
factor, a glycoprote insecreted from parietal cells of the
Ans. 1PCOS is the most common endocrine disorder of gastric fundus and body. Specific receptors in the terminal
reproductive age women. Common symptoms include ileum take up the B12 intrinsic factor complex. Vitamin
oligo- or amenorrhea, acne, hirsutism, infertility, and B12 deficiency leads to megaloblasticanemia. The patient
weight gain. Common tests used to support the diagnosis will require monthly vitamin B12 injections.
of PCOS include LH to FSH ratio, testosterone, DHEAS, and
pelvic ultrasound.
41
115. A young child is brought into the hospital present for several months and is slowly growing. He
emergency room because he has episodes of vomiting, is a nonsmoker but has a long history of chewing
headaches, problems in acquisition of motor skills, tobacco. Which of the following is the most likely to be
cranial nerve dysfunction, and problems in breathing. found on biopsy of the lesion?
This combination of syndromes most closely relates to 1. Adenocarcinoma
which of the following disorders? 2. Lymphoma
1. Cleft palate 3. Basal cell carcinoma
2. Hydrocephalus 4. Squamous cell carcinoma
3. Anencephaly
4. Syringomyelia Ans. 4. Squamous cell carcinoma is the most common
malignant tumor of the oral mucosa and may occur at any
Ans. 2. site. It most frequently involves the tongue, followed in
The symptoms described arecharacteristic of descending order by the floor of the mouth, alveolar
hydrocephalus. Hydrocephalus may come about as a result mucosa, palate, and buccal mucosa. The male to female
of defects such as the failure of formation of the cerebellar ratio is 2:1 for the gum, but 10:1 for the lip. There are
vermis, foramensof Magendie and Luschka, or of the substantial variations in the geographical distribution of
corpus callosum. There is an enlarged cranium as a result the oral cancer. For example, it is the single most common
of the buildup of cerebrospinal fluid (CSF), causing brain cancer of men in India.
damage. Several of the symptoms may also be caused by a
compression of the posterior fossa and the absence of a 118. A 25-year-old female presents for evaluation of a
cerebellar vermis. Cleft palate is a fissure of the medial breast mass and tenderness. She describes the breast
aspect of the lip and would not result in the symptoms soreness as varying with her menstrual cycles. On
described previously. Anencephaly is the complete or examination, you feel a firm, movable, irregular 2 cm
partial absence of the brain and is not compatible with life. mass. Her breast tissue is diffusely dense bilaterally.
Syringomyelia is associatedwith bilateral segmental loss of Of the options listed, which is the most likely?
pain and temperature. Acongenital aneurysm can occur in 1. Lipoma
a variety of places within the CNS and is typically 2. Fibroadenoma
associated with stroke in the adult. 3. Hemangioma
4. Intraductal papilloma
116. You are following up in your office with a patient
who had a magnetic resonance imaging (MRI) of his Ans. 2. Fibroadenoma is the most common benign
head for evaluation of persistent headaches. The MRI neoplasm of the breast and is composed of two types of
report reveals a mass suspicious for malignancy. tissues. A mesenchymal element most commonly
While explaining the need for ongoing testing to composed of edematous or collagenized fibrous tissue and
determine the etiology, the patient’s spouse asks you, an epithelial component, which consists of compressed,
“What is the most common tumor in the brain?” Which and sometimes hyperplastic, irregular ductal lumens. They
of the following would be the correct response to this are usually found in young women and may be hormonally
question? reactive during pregnancy or menopause.
1. Glioblastoma multiforme
2. Metastatic tumors 119. Skeletal Muscle relaxant which is most
3. Meduloblastoma cardiostable
4. Oligodendroglioma 1. Vecuronium
2. Atracuronium
Ans. 2. Metastatic tumors reach the intracranial 3. Cisatracurium
compartment through the blood stream, generally in 4. Pancuronium
patients with advanced cancer. Tumors of different organs
vary in their incidences of intracranial metastasis. For Ans 1. Vecuronium
example, a patient with disseminated melanoma has a SMR used in Asthmatics : Atracurium, Vecuronium.
greater than 50% likelihood of acquiring intracranial * SMR used to reduce BP : d-TC.
metastasis, whereas the incidences of such metastasis in * SMR used to maintain BP : Pancuronium.
carcinoma of the breast and lungs is 35%. Certain * SMR which is cardiostable (commonly used) :
carcinomas such as those of the prostate, liver, and Vecuronium.
adrenals and sarcomas of all types rarely establish * SMR safe in Hepatic Failure & Renal Failure : Atracurium,
intracranial metastasis. A metastasis contrast with Cisatracurium.
theprimary glioma in its discrete appearance, globoid * SMR contraindicated in Pregnancy and Renal failure :
shape, and prominent halo of edema. Gallamine.
* Pancuronium is Vagolytic and thus it can produce
117. A 60-year-old male presents to the office with a tachycardiardia.
growth on the inside of his lower lip. It has been
42
120. Which of the following is a commonly used trigeminal nerve is typical of orbital apex syndrome.
mechanism for reducing medical errors in hospitals? Proptosis and chemosis are useful signs to distinguish
1. Confidential peer review orbital apex syndrome from cavernous sinus pathology.
2. National hospital accreditation
3. Departmental grand rounds
4. Longer work shifts for employees to promote 122. A 72 year old male is being treated for
continuity of patient care hypertension, gout, Gastro-oesophageal reflux and has
a three year history of type 2 diabetes. He takes a
Ans. 1 variety of medications. His general practitioner is
Medical errors are responsible for more than 98,000 concerned after requesting U+Es on this patient which
excessive patient deaths per year. In order for medical reveal:
errors to be reduced, there need to be mechanisms for Serum Sodium 138 mmol/l
accountability which occur within a supportive Serum Potassium 4.4 mmol/l
environment. Peer review, morbidity and mortality Serum Urea 12.8 mmol/l
rounds, shorter work weeks, and root cause analysis are Serum Creatinine 162 micromol/l
all mechanisms to prevent future errors from fatigue, Of the following drugs that he takes, which one's dose
impaired system processes, and inadequate knowledge. does NOT need to be reduced for this patient?
Frequently when medical errors occur, the families want 1. Allopurinol
to know what is being done to prevent this from 2. Gliclazide
happening again. Although the peer review process is 3. Lansoprazole
confidential and not subject to subpoena, it provides an
effective mechanism to honestly evaluate our colleagues 4. Lisinopril
and enforce necessary discipline to improve patient safety.
Random drug testing is not a systemic solution. Ans. 3

121. 60-year-old woman presented with 3 months Allopurinol is useful in renal impairment, but the dose
history of diplopia and blurred vision of left eye. She should be reduced from 300mg/day to 100mg/day in
denied any pain or other neurological symptoms. Her moderate to severe renal impairment, as toxicity may
previous medical history is unremarkable. She smokes occur, leading to hypersensitivity rashes or hepatitis.
20 cigarettes per day and drinks alcohol in Gliclazide dosage should be reduced in mild renal failure,
moderation. Her general medical examination is and should be stopped in severe renal disease.
normal. Her visual acuity on the right is 6/6 and on the Lansoprazole is safe to use in renal impairment (caution in
left 6/36. liver impairment), at a dose of 15-30mg/day.
Lisinopril should be used with caution in renal
There is left partial ptosis and mild proptosis with impairment. It may potentiate hyperkalaemia and
conjunctival injection. The left pupil is smaller than hypotension, therefore the dose should be reduced to 10-
the right but reacting normally to light. There is some 20mg/day, rather than 20-40mg/day.
limitation of abduction of the left eye. Fundoscopy Metformin predisposes to lactic acidosis, therefore should
showed a pale left optic disk. The left corneal reflex is be stopped even with mild renal impairment and general
reduced. advice suggests stopping with a creatinine above 150.

The remaining of the neurological examination is 123. A 15-year-old girl is referred to clinic
normal. Routine blood tests including FBC, U+Es, LFTs, complaining of generalised muscle weakness, fatigue
TFTs, Ca, Creatine kinase, autoantibody screen were and polyuria. Her blood pressure in clinic is measured
normal. ECG, CXR were unremarkable. Slit lamp at 90/74 mmHg. Investigations:
examination was normal. Intra-ocular pressures were Serum sodium 127 mmol/l
within normal range. Where is the most likely cause of Serum potassium 3.0 mmol/l
her symptoms? Serum urea 7.2 mmol/l
Serum creatinine 110 umol/l
1. Cavernous sinus Serum chloride 92 mmol/l (NR 97-108 mmol/l)
2. Superior orbital fissure Serum bicarbonate 34 mmol/l (NR 22-28 mmol/l)
3. Orbital apex syndrome Serum magnesium 0.82 mmol/l (NR 0.8-1.1 mmol/l)
4. Optic chiasm Urine sodium 160 mmol/l (NR 40-130 mmol/l)

Ans. 3 Urine calcium 8.0 mmol/24hr (NR 2.5-8.0


mmol/24hr)
The combination of optic neuropathy, proptosis, chemosis, Which of the following is the most likely diagnosis?
horner's syndrome, ophthlmoplegia (in this case due to
6th nerve palsy), and involvement of the first branch of the
43
1. Addison's disease admission the patient had become breathless at rest
2. Bartter's syndrome and was started on oral co-amoxiclav by his general
3. Laxative abuse practitioner.
4. Liddle's syndrome On examination he was febrile 38°C and looked
unwell. Candida was noted on the tonsilar pillars. No
Ans. 2 wheeze or crackles were heard in his chest. His chest
A difficult question! This patient has a hypochloraemic radiograph is shown. Oxygen saturation was 95% on
alkalosis, with high urinary sodium and potassium loss. room air, but fell to 85% following about of coughing.
Laxative abuse tends to cause low urinary sodium and Arterial blood gases show pO2 of 59 mmHg.
potassium, with low serum bicarbonate, due to GI losses. What treatment should be given?
Thiazide diuretic abuse would fit with the serum findings, 1. Co-amoxiclav + clarithromycin
but the urinary sodium and potassium is normal (as water 2. Co-trimoxazole + prednisolone
will follow the electrolytes).
3. Vancomycin + ceftazidime
Liddle's syndrome is a congenital form of salt-sensitive 4. Cefuroxime + metronidazole
hypertension characterised by a very high rate of renal
sodium uptake, despite low levels of aldosterone, Ans. 2
secondary hypokalaemia and metabolic acidosis. It is Oral candidiasis is never normal and should always raise
caused by a congenital mutation, which causes a the possibility of AIDS and the history in this case is
constitutive hyper reactivity in the epithelial sodium characteristic of Pneumocystis carinii pneumonia. There
channel (ENaC). are several agents that can be used to treat PCP, though co-
triomoxazole is clearly the most effective and is the
The increased sodium uptake is accompanied by an treatment of first choice. Patients often deteriorate after
increased water uptake, leading to an increase in blood starting therapy for PCP as the pneumonitis worsens due
volume, and secondary hypertension. to the inflammation associated with dying pneumocysts. In
patients with a pO2<60 mmHg, oral prednisolone is added
Addison's disease is primary adrenal insufficiency. The to reduce the inflammatory effect.
most common cause in the western world is autoimmune
adrenalitis, and worldwide causes include TB, fungal 125. A 26 year old female presents with a six weeks
infections (histoplasmosis, cryptococcus) and history of galactorrhea. She has no other symptoms
cytomegalovirus. Patients with Addison's disease have but takes medication for contraception, dyspepsia and
glucocorticoid and mineralocorticoid deficiency. This leads migraine. Examination reveals slight galactorrhea
to hypotension, hyponatraemia (in 90%) and with expression from both breasts but is otherwise
hyperkalaemia (in 65%). normal.
Investigations show:
Bartter's syndrome is a rare, autosomal recessive disorder, Prolactin 915 mU/L (<450)
caused by one of three mutations of the ion transporter or Which one of the following drugs may be responsible?
ion channel present in the thick ascending limb of the 1. Codeine phosphate
distal nephron. Type I and II mutations present in infancy 2. Metoclopramide
(often following premature birth and polyhydramnios) 3. Omeprazole
with severe dehydration, hypokalaemic alkalosis, 4. Oral contraceptive pill
hypercalciuria and nephrocalcinosis.
Ans. 2
Mortality is high. Type III mutations present with a more Metoclopramide acts as a dopamine antagonist. Dopamine
varied clinical picture to type I and II, ranging in severity inhibits the release of Prolactin from the anterior Pituitary
from near fatal volume depletion with hypokalaemic gland. Therefore ,metoclopramide can predispose to
alkalosis and respiratory arrest, to mild disease presenting hyperprolactinemia and consequent galactorrhea.
in teenagers with weakness and polyuria.
Nephrocalcinosis has not been described in type III 126. A 70 year old female is admitted acutely unwell. 6
mutations, therefore it can differentiate between type I weeks prior to admission she had presented to the GP
and II disease, and type III disease. Management is with with tiredness and weight loss and had been
long term potassium supplementation and care to avoid diagnosed with hypothyroidism based on results
dehydration. The long term prognosis is uncertain. which show a T4 of 7.8 nmol/l (9-22) and a TSH of 4
mu/l (0.5-4.5). She was treated with thyroxine 100
micrograms daily and has since deteriorated. She has
124. A 32 year-old man presented to hospital with a no other past medical history of note, does not smoke
four week history of progressively worsening and drinks modest quantities of alcohol. She is a
dyspnoea on exertion. He also complained of a non- widow and is self caring. Her mother had
productive cough. Over the two days preceeding hypothyroidism.
44
On examination, she is drowsy, thin, has a temperature associated with hyperparathyroidism, but more recent
of 37.5oC, a pulse of 98 beats per minute and a blood evidence questions this observation.
pressure of 84/50 mmHg. Cardiovascular, respiratory
and abdominal examination are otherwise normal. 128. A 28 year old woman without any past medical
There are no neurological abnormalities. The house history presents with a 3 month history of arthralgia.
officer has sent some emergency bloods on this She had no past medical history of note. Examination
patient. Whilst awaiting the results, what is the most reveals swelling of the distal interphalangeal joints of
appropriate immediate treatment for this patient? the middle and ring fingers of the hand and wrist on
1. IV 50% Dextrose the right plus a swollen left ankle. Investigations show:
2. IV Cefotaxime ESR 40 mm/hr (0-10)
3. IV Hydrocortisone Which of the following is the most likely diagnosis?
1. Acute exacerbation of osteoarthritis
4. IV Thyronine (T3) 2. Psoriatic arthropathy
3. Rheumatoid arthritis
Ans. 3 4. Reactive arthritis
The specific features in this case are the past history of
weight loss associated with a low T4 but normal TSH. The Ans. 2
latter would suggest either sick euthyroidism which can be This woman has psoriatic arthritis. Synovitis is indicative
associated with hypoadrenalism or secondary of an inflammatory arthritis. Rheumatoid arthritis typically
hypothyroidism. In hypoadrenalism which is either effects the metacarpophalangeal and proximal
primary or secondary, the addition of thyroxine can interphalangeal joints symmetrically. Psoriatic arthritis
precipitate acute hypoadrenalim which appears to be the effects the distal interphalangeal joints and tends to be
case here. The use of IV steroids is therefore life saving and asymmetrical. Joint involvement in SLEoccurs in the form
should be given without hesitation. of a polyarticular arthralgia, frequently symmetrical and
episodic. Intense tendonitis is more common than
127. A 73-year-old woman presented with thirst and synovitis and can lead to deforming reversible subluxation
polyuria of six months duration. She had suffered of joints without erosive disease (Jaccoud's arthropathy). A
several episodes of lower back pain. She was on no short, striking history of marked, acute polyarticular
medication. On examination she looked well, had a symptoms occurs with systemic (viral) infection.
dorsal kyphosis and a blood pressure of 170/95
mmHg. Investigations revealed: 129. A 32-year-old man is referred to the general
erythrocyte sedimentation rate 15 mm/1st hour (0- medical outpatient clinic. He and his wife had been
30) referred by his GP to the infertility clinic for
serum urea 11.9 mmol/L (2.5-7.5) consideration for assisted conception. He is generally
serum creatinine 175 umol/L (60- fit and well and works as a central heating engineer.
110) He has been referred because he has a chronic
serum albumin 40 g/L (37-49) productive cough and a history of recurrent chest
serum total calcium 2.98 mmol/L (2.2-2.6) infections since childhood in addition to recurrent
What is the most likely cause of the Hypercalcaemia? sinusitis. Investigations show: Sodium sweat test:
Normal Immunoglobulins (IgG, IgM, IgA): Normal What
1. Metastatic breast cancer is the most likely diagnosis?
2. Myeloma 1. Bronchiectasis
3. Osteoporosis 2. Chediak-Higashi syndrome
4. Primary hyperparathyroidism 3. Cystic fibrosis
4. Primary ciliary dyskinesia

Ans. 4
Comments: The prevalence of hyperparathyroidism is Ans. 4
said to be 4 per 1000 in women over 60, and is 2-3 times Primary ciliary dyskinesia is inherited as an autosomal
more common in women than men. The lower back pain recessive disorder characterized by abnormal ciliary
may be loin pain due to renal colic caused by renal calculi. motion and impaired mucociliary clearance. The lack of
The renal impairment may be associated with renal calculi, effective ciliary motility, causes abnormal mucociliary
perhaps due to calculi-induced hydronephrosis in extreme clearance. This leads to recurrent or persistent respiratory
cases. Renal impairment in hyperparathyroidism is not infections (which may lead to bronchiectasis), sinusitis,
uncommon. Chronic hypercalcaemia can compromise the otitis media, and male infertility.
renal concentrating ability, leading to polydipsia and
polyuria. The kyphosis may be due to osteoporosis, which In 50% of the patients, PCD is associated with situs
is commonly seen in conjunction with inversus (Kartagener's syndrome). The principal
hyperparathyroidism. Classically, hypertension has been differential diagnoses to consider in this case are cystic
45
fibrosis and primary ciliary dyskinesia. The diagnosis of CF which had developed over a six month period with
is based on typical pulmonary and/or gastrointestinal symptoms being entirely relieved by eating. She had
tract manifestations and positive results on sweat test developed one of these episodes whilst on the ward
(pilocarpine iontophoresis). A negative sweat test is and a BM monitor showed a value of 2 mmol/l. She
sufficient evidence to exclude CF as a diagnostic took some glucose tablets and had quickly recovered.
possibility. While the patient may have bronchiectasis, this On examination no specific abnormalities were found
alone is not sufficient to account for his other symptoms. with a blood pressure of 118/74 mmHg, a pulse of 72
beats per minute and a BMI of 22 kg/m2.
130. A 80 year old female presents to A+E acutely She was admitted for a 72 hour fast and at 3 am, 16
unwell after being found unresponsive on the floor of hours into the fast she develops typical symptoms. Her
her house by her son. She had a past history of BM is measured at 2.2 mmol/l, the fast is stopped and
hypothyroidism and taking thyroxine daily. However, bloods taken. The following reveal her results:
her compliance with treatment is questionable. On Plasma Glucose 1.8 mmol/l (3 – 6)
examination she was unrouseable with a Glasgow Plasma 3Beta-HydroxyButyrate 0.5 mmol/l (greater than
Coma Scale of 6/15, had a central temperature of 34oC, 1 mmol/l)
Oxygen saturations of 95% on air, a pulse of 44 beats Plasma Insulin 450 pmol/l (less than
per minute and a blood pressure of 100/80 mmHg. 21)
There were no specific localizing signs on neurological C-peptide 0.2 nmol/l (less than 0.5)
examination but both plantars were extensor. Prior to What is the most likely diagnosis?
results of her emergency blood tests being available, 1. Adult Glycogen storage disease
what is the most appropriate immediate treatment for 2. Factitious hypoglycemia due to Insulin treatment
this patient? 3. Factitious hypoglycemia due to Sulphonyl urea
1. Intravenous 50% Glucose treatment
2. Intravenous Hydrocortisone 4. Insulinoma
3. Intravenous Normal Saline
4. Intravenous Thyronine (T3) Ans. 2
This patient has developed hypoglycaemia with
suppression of her 3betaHydroxybutyrate (a ketone),
Ans. 4 elevated insulin yet suppressed C-peptide. This would
This patient has typical features of Myxoedema coma suggest that there is insulin induced hypoglycaemia and as
which usually occurs in the elderly who are typically non- C-peptide is suppressed indicates exogenous
compliant. It carries a high mortality and should initially administration of insulin. Sulphonylureas would produce
be treated with IV thyroid hormone – either T4 or T3 – raised insulin and C-peptides and could be assessed in
even before results are obtained. Steroids are often suspicious cases by measuring a sulphonylurea
employed, in case of associated hypoadrenalism but in this concentration. Again insulinoma would be associated with
case would probably not be the most immediate choice. proportionately elevated insulin and C-peptide.
Other treatment includes rewarming. IV fluids should be
used cautiously as these patients are typically fluid 133. A 55 year old female presents with episodic
overloaded. sweats and tremors which are relieved by glucose. She
has gained approximately 6 kg in weight of late and
131. Infant circuit for anaesthesia - drinks approximately 10 units of alcohol weekly. Her
1. Bains circuit investigations show normal Full Blood Count, Normal
2. Magill circuit Urea and electrolytes and a fasting plasma glucose
3. Ayres t piece concentration of 4 mmol/l (3-6). What is the most
4. Water's circuit appropriate investigation for this patient?
1. 72 hour fast
Ans. is ‘3’ i.e., Ayres t piece 2. CT scan of pancreas
Circuit of choice for spontaneous ventilation in 3. EEG
adultMapleson A 4. Insulin and C-peptide concentration
Circuit of choice for controlled ventilation in
adultMapleson D (Bain circuit) Ans. 1
Circuit of choice for childrenType F, i.e. Jackson-Rees This patient describes symptoms suggestive of
(first choice) and hypoglycaemia which are relieved by carbohydrate. The
TypeE, i.e. Ayres Tpiece (secondchoice) likely cause is an insulinoma which is producing the
weight gain. The diagnosis is made by demonstration of
132. A 33 year old nurse is admitted for prolonged inappropriately high insulin and C peptide during
fasting. She originally presented to clinic with a spontaneous hypoglycaemia.
history of episodic sweating and light-headedness

46
134. A 62-year-old male attends A+E with a severe hands of farmers, but does not fully explain this clinical
nose bleed. He is known to have alcoholic cirrhosis. picture. Stevens-Johnson syndrome typically starts with an
His investigations reveal: erythema multiforme-type rash that spreads widely and
haemoglobin 10.9 g/dL (13.0-18.0) involves the buccal mucosa and conjunctivae; often caused
white cell count 5 x 109/L (4-11) by antibiotic therapy, but does not fit the clinical picture
platelet count 60 x 109/L ( 150-400) presented here.
prothrombin time 17.5s (11.5-15.5)
APPT 42s (30-40) 136. A 42-year-old HIV-seropositive man presents to
fibrinogen 0.7 g/L (1.8-5.4) Casualty with a two-week history global headache. His
What is the most appropriate blood product for this partner says that he has become increasingly confused
patient? and disorientated. The patient's latest CD4 count,
1. Cryoprecipitate taken three weeks ago, was 50 cells/mm3. He had
chosen not to take antiretroviral therapy, but was
2. Factor VIII taking co-trimoxazole as prophylaxis against
3. Platelets Pneumocystis carinii pneumonia. On examination he
4. Prothrombin complex concentrate had mild weakness of his left arm and leg in all muscle
groups and a right homonymous hemianopia.
Fundoscopy was normal with no evidence of
Ans. 1 papilloedema. A CT scan of his brain showed several
The most significant abnormality is the low Fibrinogen. areas of low attenuation in both cerebral hemispheres,
Therefore the best product to correct the fibrinogen out of but there was no enhancement with contrast and no
those given is cryoprecipitate. To correct a coagulopathy mass effect. What is the most likely diagnosis?
you need to aim for Fibrinogen > 1.0g/l, Platelets >50 x
109/L, PT and APTT < 1.5 upper range of normal- so from 1. Cerebral lymphoma
the results you can see the most significant abnormality is 2. Cerebral toxoplasmosis
the low fibrinogen, the platelets are low and APTT/PT 3. HIV encephalopathy
prolonged but not really sufficient to cause bleeding. 4. Progressive multifocal leukoencephalopathy

135. A 40-year-old farmer presented to Casualty with a Ans. 4


24-hour history of fever and increasing confusion. On The most likely diagnosis is Progressive multifocal
examination he was febrile 39.5‫؛‬C. A generalized leukoencephalopathy (PML), a demyelinating disease seen
erythematous rash, covering most of his body, was in advanced HIV/AIDS and caused by the JC virus. Cerebral
observed. He also had a paronychial infection of his lymphoma and cerebral toxoplasmosis are often
right index finger, with lymphangitis extending associated with mass effect on CT brain scanning. In CNS
caudally and with axillary lymphadenopathy. His heart lymphoma there is usually a solitary lesion. Cerebral
rate was measured at 120 beats per minute with a toxoplasmosis is frequently associated with multiple
blood pressure of 80/60 mmHg. What is the most lesions that show ring enhancement with iv contrast. HIV
likely diagnosis? encephalopathy may be associated with confusion, but is
1. Hantavirus infection not associated with this CT appearance. This is not a
2. Leptospirosis typical presentation of neurosyphilis in any of its forms.
3. Orf
4. Staphylococcal toxic shock syndrome 137. A 54 year-old male presents with progressive
pins and needles and numbness in both feet which
Ans. 4 have deteriorated over the last six months. He has a 10
The history is typical of staphylococcal toxic shock year history of Type 2 diabetes mellitus and had
syndrome (TSS) – shock, fever, confusion and rash. The cervical spondylosis for which he underwent surgery 8
primary source of infection in this case is the paronychia of years ago. He also confessed to drinking
his right index finger. Hantavirus infections (viral approximately 40 units of alcohol weekly.
zoonoses transmitted via rodents) typically have two On examination he had a mild bilateral weakness of
distinct presentations: either as a haemorrhagic fever with foot dorsiflexion, both ankle reflexes were absent and
renal failure or as an acute pulmonary syndrome. The plantar responses were flexor. There was absent
former manifestation is commonest in the far east and sensation to light touch to mid-shin level with loss of
eastern Europe, while the latter is the predominat form in joint position sensation in the toes and absent
the southwestern United States and South America. vibration sensation below the hips. He had a marked
Leptospirosis does not typically cause a rash and is often sensory ataxia and pseudoathetosis of upper limbs. He
associated with jaundice; leptospirosis would not explain had no evidence of a retinopathy and urinalysis was
the lesion on his finger. Orf, a zoonotic infection caused by normal. What is the most likely diagnosis?
a pox virus, presents with painless ulcerated lesions on the 1. Alcohol-induced neuropathy
47
2. Central lumbar disc prolapse Investigations: Bone mineral density (BMD) was
3. Cervical cord compression assessed by dual-energy x'ray absorptionmetry at the
4. Vitamin B12 deficiency hip and the lumber spine (L1-L4).
BMD T-score at lumber spine: -3.0
BMD T-score at total hip: -2.8
Ans. 4 What is the diagnosis?
Diabetic peripheral neuropathy usually goes in parallel 1. Osteopaenia
with retinopathy and nephropathy. It is also slowly 2. Osteomalacia secondary to Vitamin D or calcium
progressive and affects mainly spinothalamic pathway. deficiency
Alcohol induced peripheral neuropathy is also slowly 3. Post menopausal osteoporotic vertebral fractures
progressive and affects mainly spinothalamic pathway.
Vitamin B12 deficiency usually causes a more rapidly 4. Senile osteoporosis
progressive neuropathy with dorsal column involvement
(joint position and vibration involvement with sensory Ans. 3
ataxia and pseudoathetosis of upper limbs). This lady has several risk factors for post-menopausal
osteoporosis, including early onset (<45 years)
138. A 44 year old male attends for a health check at a menopause, absence of hormone replacement therapy,
mobile cardiovascular risk assessment clinic. He takes calcium and Vitamin D supplemention and low body
no medication but leads a sedentary lifestyle. He is a weight. Patients with osteoporosis may have no warning
non-smoker and family history reveals that his father signs until the first fracture occurs. Gradual height loss and
had an MI at 60 years old. The results of his lipid dorsal kyphosis may result from microfractures or
investigations reveal: complete fractures of vertebral bodies.
Total Cholesterol 5.0 mmol/l (less than 5.5) A T-score refers to the difference in standard deviation
Triglycerides 4.0 mmol/l (1- 2.2) (SD) from the mean bone mass. The World Health
Which of the following is the commonest cause of an Organisation (WHO) criteria for the diagnosis of
isolated hypertiglycerdemia? osteoporosis is as follows:
1. Alcohol
2. Diabetes Mellitus Diagnosis
3. Drug Therapy T-Score Normal, > or = to -1.0
4. Obesity Osteopaenia -1.0 to -2.5
Osteoporosis < or = to -2.5
Causes of secondary osteoporosis such as osteomalacia,
Ans. 4 bone metastasis, myeloma must be excluded when a
Comments: vertebral fracture is found.
The commonest cause of a mild hypertriglyceridaemia is
obesity secondary to a reduced efficacy of lipoprotein 140. A 65 year old male is admitted to the coronary
lipase activity and overproduction of VLDL. Obesity care unit with an acute inferior myocardial infarction.
(defined as a BMI above 30) is present in approximately There are no contra-indications to thrombolysis and
20% of subjects in the UK and rising and hence this is why he receives streptokinase with good resolution of ECG
it is the commonest cause of hyperlipidaemia. Alcohol is changes. Three days later examination is normal, with
probably a close second. Other secondary causes of a blood pressure of 134/76 mmHg. Results reveal a
hypertriglyceridaemia include pregnancy, hypothyroidism, total cholesterol of 4.8 mmol/L (normal <5.2).Which
diuretics and pancreatitis. one of the following drugs does not have good
evidence for reducing future morbidity and mortality?
139. A 42 year old female presented with acute back 1. Aspirin
pain after lifting a chair. The pain is intense, increases 2. Atenolol
on movement, and radiates bilaterally to the 3. Simvastatin
hypochondriacal region. Upon questioning she denied 4. Nifedipine
previous fractures, but admitted to a gradual loss of
height (5cm from her young adult height) and Ans. 4
occasional self-limiting back pain. Aspirin leads to a 12% reduced risk of death and 31%
Past medical history included spontaneous reduced risk of reinfarction in evidence reviewed by the
menopause at the age of 37. She had never taken any Antiplatelet therapy trialists and also GISSI studies. Several
regular medications in the past, calcium or Vitamin D trials have demonstrated benefit from long term treatment
supplements. with beta blockers, by reducing the incidence of recurrent
Examination revealed a thin woman who had a dorsal MI, and death from all causes.
hyperkyphosis. Severe back pain was ellicited on Numerous trials have shown benefit from ACE inhibitor
movement and local percussion. therapy post MI in those with and without evidence of left
ventricular impairment.
48
The 4S (Scandinavian Simvastatin Survival Study) non invasive and will give a diagnosis. The patient may
demonstrated a benefit from lowering cholesterol with have lymphadenopathy, splenmegaly- which would show
Simvastatin in patients with coronary disease. There is no on CT, but no diagnosis can be made from this. A bone
evidence to support a beneficial effect of nifedipine post marrow is invasive, and the BM is sometimes not involved
MI. in low grade lymphoproliferative disorders and similarly
there may be no cytogenetic abnormality. The FBC is not
141. A 50 year old woman complains of arthritis and suggestive of TB or malignancy, therefore sputum
swelling of approximately 4 months duration. On examination would not be useful.
examination, she has a symmetrical inflammation with
painful movements of the hands and feet and also
swelling of both knees, suggesting a diagnosis of 143. A 45 year old male diabetic is seen in clinic at the
rheumatoid arthritis. Regarding her joint disease, diabetic clinic for annual review. He has been a
which of the following suggest an adverse prognosis? diabetic for 8 years and attends clinic regularly. He is
also being treated for hypertension and takes
1. Metastatic disease metformin 500mg tds, gliclazide 80mg daily,
2. Polymyalgia rheumatica atorvastatin 10mg/d, ramipril 10mg/d and
3. Spinal stenosis bendrofluazide 2.5 mg/d. The registrar notes that he
4. Spondylosis of the lumbar spine has central obesity and some thin skin and arranges
tests for Cushing's syndrome. He is seen three months
later in clinic, is seen by another registrar who does
Ans. 4 not think that he has any features to suggest Cushing's
Articular erosions in rheumatoid arthritis occuring early syndrome and has the results of the previously
on in the course of the disease, especially within the first 6 requested investigations:
months of presentation, indicate a poor prognosis. Over HbA1c 8.1% (<6.5)
time joint damage will relate to disability. A positive Fasting glucose 9 mmol/l (3.5-6)
rheumatoid factor is associated with more severe erosive 24hr Urine free cortisol 354 mmol/d (<250)
disease, extra-articular manifestations including 9am Plasma ACTH concentration 12 ng/dl (10 –50)
subcutaneous nodules and increased mortality. An acute CT abdomen 3cm right sided adrenal
onset of presentation is not a poor prognostic factor. mass
Raised inflammatory markers (CRP, ESR) and the duration What is the adrenal mass?
of the early morning stiffness both correlate with disease 1. Aldosterone secreting adenoma
activity. 2. Cortisol secreting adenoma

142. A 72 year old male presents with a five day 3. Ectopi CRF producing Phaeochromocytoma
history of cough, dyspnoea and fever. His chest X-ray 4. Incidentaloma
shows a left basal consolidation. His Full Blood Count
shows: Ans. 2
Haemoglobin 11 g/dL (13.0-16.5) The patient is likely to have Cushing's syndrome as
White cell count 30 x 109/L (4-11) suggested by the elevated UFC and the lowish ACTH plus
Neutrophils 10 x 109/L (2-7) adrenal mass on CT would suggest that this is due to a
Lymphocytes 20 x 109/L (1-4) functional adrenal adenoma. The absence of physical signs
Monocytes 1 x 109/L (0-0.8) of Cushing's syndrome
Eosinophils 0.4 x 109/L (0.04-0.4)
Basophils 0.1 x 109/L (0-0.1) 144. A 50 year old woman with a long history of
Which one of the following is the most appropriate test alcohol abuse is prescribed Phenytoin for epilepsy.
to establish the diagnosis? Examination was normal except for a liver edge. Her
1. Bone marrow aspirate full blood count reveals
2. Bone marrow cytogenetics hemoglobin 10.0 g/dL (13-18)
3. CT abdomen MCV 122 fL (80-96)
4. Immunophenotyping of white cells White cell count 2.2 x 109/L ( 4-11)
Platelet count 85 x 109/L ( 150-400)
What is the most likely explanation for these results?
Ans. 4 1. Alcoholic liver disease
Apart from the mild neutrophilia, which could be 2. Aplastic anaemia
explained by the infection, the significant abnormality on 3. Folic acid deficiency
the FBC is the lymphocyte count. Such a high lymphocyte
count could be suggestive of a lymphoproliferative 4. Hypothyroidism
disorder such as chronic lymphocytic leukaemia. The best
way to diagnose these is immunophenotyping of the blood- Ans. 3
49
The history of episodic headaches is central to this
Folic acid deficiency would give all these results. In question, together with the tachycardia. These paroxysmal
addition she has good reason to be folate deficient- drinks headaches suggest the diagnosis of phaeochromocytoma;
a considerable amount and is on anticonvulsants. often the symptoms are vague, and rarely is the classical
Alcoholic liver disease, on its own would not make you presentation encountered. Patients with
leucopenic. Hypothyroidism, would cause a raised MCV, pheochromocytoma may develop a severe vascular
but not the other parameters. Scurvy does not cause this headache (Bridgwater and Starling, 1982). Thomas et al.
picture. Aplastic anaemia could cause this haematological (1966) reviewed the histories of 100 patients with proven
picture, but the clinical scenario leads you towards folic pheochromocytoma seen at the Mayo Clinic and found that
acid deficiency. episodic headache was present in 80%. It was usually of
rapid onset, bilateral, severe, throbbing, and associated
145. A 38 year old man presented with intermittent with nausea in about half of the cases.
severe headaches. He was prescribed Spironolactone
50mg and Bendrofluazide 2.5mg daily for
hypertension. On examination his pulse was 112 beats 146. A 30-year-old man presented with a painless
per minute, with regular rhythm, and blood pressure visual loss of his left eye over 24 hours. In two weeks
was 190/110 mmHg. Investigations revealed: he lost the vision of his right eye. There has not been
serum sodium 132 mmol/L (137-144) any improvement of his visual acuity over the next 2
serum potassium 3.4 mmol/L (3.5-4.9) months. He denied any other symptoms. His previous
serum urea 7.0 mmol/L (2.5-7.5) medical history is unremarkable. He smokes 10
Which one of the following is the most useful cigarettes per day and drinks 20 units of alcohol per
investigation in establishing the diagnosis? week. There is no relevant family history. He does not
1. A 24 hour urinary 5-hydroxyindoleacetic acid take any drugs. General medical examination is
concentration normal. His visual acuity is 6/60 on the right and
2. A 24 hour urinary catecholamine concentration finger counting on the left. His colour vision is
impaired in both eyes. He has bilateral optic atrophy.
3. A 24 hour urinary free cortisol concentration The remaining of his neurological examination is
normal. Blood tests including FBC, U+Es, LFTs, TFTs,
4. A radionuclide hippuran renogram Ca, glucose, autoantibody screen, VDRL, Vit B12 are
normal. CXR is normal. MRI brain and orbits are
normal. CSF analysis is unremarkable. What is the
Ans. 2 most likely diagnosis?
This question is typical of an MRCP question which may be 1. Multiple sclerosis
used as a "good discriminatory question"! It seems 2. Alcohol/tobacco induced optic neuropathies
nebulous at first, but on further investigation it is
extremely complex. One can imagine the smirks of the old 3. Leber's optic atrophy
gentlemen in the college at the post-exam port soiree.
4. Giant cell arteritis
The answer is unlikely to be carcinoid given the lack of
symptoms of carcinoid syndrome. The flushing attacks of Ans. 3
the carcinoid syndrome are accompanied by hypotension. Leber's optic atrophy usually affects young men. It causes
Renal anatomy and function is studied with sequential sequential optic neuropathies in days to weeks. It is typical
images using radionuclides that are indexes of tubular painless and severe. Visual acuity fails to improve. Optic
function (131-I Hippuran). The clinical history here is not neuritis is usually painful, and visual acuity improves over
suggestive of renal artery stenosis. Given the patients' a matter of weeks. Giant cell arteritis affects elderly
young age, and markedly raised BP on treatment, we patients. Alcohol/Tobacco optic neuropathies are usually
should consider an endocrine cause. chronic.

The electrolyte disturbance is mild, and is of dubious 147. A 69 year-old man is admitted with pain and
relevance in this question. Diuretic use may be causing the numbness in his right foot, following a right hip
hyponatremia and hypokalemia, indeed the commonest replacement. In his past medical history he had been
cause of hypokalemia in hypertension is diuretic therapy. treated for lower backache by his GP. On examination,
However, spironolactone use could, theoretically, mask a there was weakness of all movements at the right
more significant hypokalemia. There is no clinical history ankle, with absent right ankle jerk, and sensory
to suggest Cushing's, and primary aldosteronism is not impairment on the lateral aspect and sole of the foot.
associated with a tachycardia. An aldosterone;rennin ratio Where is the most likely site of the lesion?
would not be appropriate at this stage given that the 1. Femoral nerve
patient is receiving spironolactone. 2. Lumbosacral plexus
3. Obturator nerve
50
4. Sciatic nerve self-limiting viral illness, parvovirus B19 infection may
have more serious sequelae. The virus has a tropism for
rapidly dividing erythrocyte precursors which they infect
Ans. 4 and destroy. Thus, no reticulocytes (immature
Sciatic nerve palsy is a known complication of a total hip erythrocytes) are available to replace aging or damaged
replacement (as femoral nerve palsy). It causes global erythrocytes as they are cleared by the reticuloendothelial
weakness of the ankle due to the involvement of both of its system. This may not have any significant impact on
branches: tibial nerve (plantaflexion and inversion) and otherwise healthy individuals, but can trigger an aplastic
common peroneal nerve (dorsiflexion and eversion). The crisis – particularly in patients with haemoglobinopathies.
left ankle jerk is absent due to tibial nerve involvement Parvovirus B19 infection may also be associated with a
symmetrical post infectious arthritis, affecting the small
148. An 18 year old female presents with an acute joints of the hands and feet.
exacerbation of asthma associated with a chest
infection. She is unable to complete a sentence and her The knees or elbows are rarely involved. The arthritis is
peak flow rate was 35% of her normal level. She is much more common in adults, particularly in women, and
treated with high flow oxygen, nebulized may persist for weeks to months (even years in a small
bronchodilators and oral steroids but this is number of patients). The arthritis may mimic rheumatoid
associated with little change in her condition. Which of arthritis. Unlike rheumatoid arthritis, the post infectious
the following treatments, given intravenously, would arthritis associated with parvovirus B19 does not cause
be the most appropriate for this patient? permanent damage to bones or joints.
1. Aminophylline
2. Augmentin 150. A 17 year old girl is admitted with a 2 day history
3. Hydrocortisone of rigors due to a urinary tract infection. On
4. Magnesium examination she appears unwell, has a Body Mass
Index of 31kg/m2, a temperature of 39°C; examination
is otherwise normal. Initial biochemistry revealed:
Ans. 4 Potassium 4 mmol/L (3.5-5)
This patient has acute severe asthma (PEFR 33-50% Urea 7 mmol/L (2.5-7)
predicted/best, unable to complete sentences in one Glucose 33 mmol/L (3.0-6.0)arterial
breath, respiratory rate > 25 breaths/min, pulse rate > 110 blood gases
beats/minute). She has not responded to initial treatment pH 7.3 (7.36-7.44)
and the treatment of choice now is intravenous Standard bicarbonate 14 mmol/l
magnesium 1.2 – 2g over 20 minutes. Base deficit -10
urinalysis negative for ketones Which one of the
149. A 22-year-old nursery school teacher is referred following is the best initial treatment for her
to clinic by her GP. She gives several weeks' history of hyperglycaemia?
swollen, tender finger joints. She has no past history of 1. Metformin
note and has been entirely well apart from a recent 2. Metformin plus Gliclazide
viral infection. She had no significant travel history. 3. Rosiglitazone
The only sick contacts she could recall were several of 4. Sliding scale IV insulin infusion
the children in her class who had recently been off sick
with a mild self-limiting illness consisting of a fever,
rhinorrhoea and a rash. On examination the Ans. 4
metocarpo-phalangeal joints and proximal
interphalangeal joints of both hands are swollen and Comments: This patient has a metabolic acidosis with pH
tender. What is the most likely diagnosis? of 7.3 and low bicarbonate likely due to sepsis. She is a
1. Adult Still's disease type 2 diabetic with uncontrolled hyper glycaemia but is
2. Lyme disease unlikely to have diabetic ketoacidosis because the urine is
3. Parvovirus B19 infection negative for ketones. It is important that her glycaemia is
controlled to promote recovery from the sepsis this is best
4. Rheumatoid arthritis achieved with intravenous insulin initially.

Ans. 3
Parvovirus B19 is a single-strand DNA virus. The most
widely known clinical manifestation of parvovirus B19 is
erythema infectiosum ('slapped cheek syndrome'), a mild
viral illness of childhood characterized by a classic
exanthem in which both cheeks appear bright red as
though they had been slapped. Although usually a benign
51

Potrebbero piacerti anche